Final Exam Review

Lakukan tugas rumah & ujian kamu dengan baik sekarang menggunakan Quizwiz!

Client is on treatment for RA for 3 weeks. During administration of etanercept, what is most important for the nurse to assess? (Exam 4)

The white blood cell counts and platelet counts

A nurse is assessing a client who underwent abdominal surgery 10 days ago. The client complains of pain in the abdomen. What type of pain does the client experience? (Exam 5)

visceral pain

Which finding can indicate gallstones and gallbladder obstruction? (Exam 5)

referred pain to the right shoulder

A client has a tentative diagnosis of primary biliary cirrhosis. What skin change does the nurse expect to observe when performing a physical assessment? (Exam 5)

Telangiectasia

The nurse is providing discharge teaching for a client with newly diagnosed Crohn's disease about dietary measures to implement during exacerbation episodes. Which statement made by the client indicates a need for further instruction? (Exam 5)

"I should increase the fiber in my diet"

The primary health care provider has determined that a client has contracted hepatitis A based on flu-like symptoms and jaundice. Which statement made by the client supports this medical diagnosis? (Exam 5)

"I ate shellfish about 2 weeks ago at a local restaurant."

A client is being discharged to home after application of a plaster leg cast. Which statement indicates the client understands proper care of the cast? (Exam 4)

"I need to avoid getting the cast wet"

The nurse is giving instructions to a client who sustained a ligament injury who is returning home after a knee arthroscopy. Which statement by the client indicates that the instructions are understood? (Exam 4)

"I need to report a fever, redness around my incisions, or persistent drainage to my health care provider."

The nurse has given the client instructions about crutch safety. Which statements indicate that the client understands the instructions? (Exam 4)

"I would not use someone else's crutches" "I need to remove any scatter rugs at home" "I need to have spare crutches and tips available"

The registered nurse instructs the new nurse in orientation regarding the physiologic processes of the endocrine system prior to client assessment. Which statement made by the new nurse indicates effective learning? (Exam 5)

"The hormones of the endocrine system exert their action by 'lock and key' mechanism

A patient has a normal cardiac rhythm and a heart rate of 72 beats/minute. The nurse determines that the P-R interval is 0.24 seconds. The most appropriate intervention by the nurse would be to (Exam 2) a. notify the health care provider immediately. b. give atropine per agency dysrhythmia protocol. c. prepare the patient for temporary pacemaker insertion. d. document the finding and continue to monitor the patient.

. d. document the finding and continue to monitor the patient.

A registered nurse is teaching a nursing student about the proper techniques of an abdominal assessment. Which order of assessment stated by the nursing student indicates effective learning? (Exam 5)

1. "I will inspect the surface motion of the abdomen" 2. "I will note the position of the umbilicus" 3. "I will assess for bowel motility by auscultation" 4. "I will palpate to assess for any abdominal tenderness"

The nurse recalls that the events of heat production through the endocrine system occur in what order? (Exam 5)

1. Hypothalamus secretes thyrotopin-releasing hormone 2. Stimulation of the anterior pituitary gland 3. Release of thyroid-stimulating hormone 4. Release of thyroxine from the thyroid gland 5. Release of epinephrine from activated adrenal medulla 6. Increase in metabolic rate, stimulation of glycosis, and vasoconstriction

In monitoring a client's response to disease modifying antirheumatic drugs, which assessment findings are acceptable responses? (SATA) (Exam 4) 1. Symptom control during periods of emotional stress 2. Normal white blood cell, platelet, and neutrophil counts 3. Radiological findings that show no progression of joint degeneration 4. An increased range of motion in the affected joints 3 months into therapy 5.Inflammation and irritation at the injection site 3 days after the injection is given 6. A low-grade temperature on rising in the morning that remains throughout the day

1. Symptom control during periods of emotional stress 2. Normal white blood cell, platelet, and neutrophil counts 3. Radiological findings that show no progression of joint degeneration 4. An increased range of motion in the affected joints 3 months into therapy

A client receiving Digoxin daily has a potassium of 3 and reports anorexia. The cardiologist prescribes a serum digoxin level to be done. Which level would the nurse recognize as being outside of the therapeutic range?

2.2 ng/mL

The nurse is conducting health screening for osteoporosis. Which client is at greatest risk is developing this problem? (Exam 4)

A sedentary 65-year-old client who smokes cigarettes

The nurse assumes care of a patient who just returned from surgery for a total laryngectomy and radical neck dissection and notes the following problems. In which order should the nurse address the problems? (Put a comma and a space between each answer choice [A, B, C, D].) a. The patient is in a side-lying position with the head of the bed flat. b. The patient is coughing blood-tinged secretions from the tracheostomy. c. The nasogastric (NG) tube is disconnected from suction and clamped off. d. The wound drain in the neck incision contains 200 mL of bloody drainage. (Exam 1)

A, B, D, C The patient should first be placed in a semi-Fowler's position to maintain the airway and reduce incisional swelling. The blood-tinged secretions may obstruct the airway, so suctioning is the next appropriate action. Then the wound drain should be drained because the 200 mL of drainage will decrease the amount of suction in the wound drain and could lead to incisional swelling and poor healing. Finally, the NG tube should be reconnected to suction to prevent gastric dilation, nausea, and vomiting.

When anazlysing an EKG rhythm strip of a patient with a regular heart rhythm, the nurse counts 30 small blocks from one R wave to the next. The nurse calculates the patient's heart rate as: 50 When preparing to defibrillate a patient. In which order will the nurse perform the following steps? (Put a comma and a space between each answer choice [A, B, C, D, E].) (Exam 2) a. Turn the defibrillator on. b. Deliver the electrical charge. c. Select the appropriate energy level. d. Place the paddles on the patient's chest. e. Check the location of other staff and call out "all clear."

A, C, D, E, B

The nurse is caring for a patient who has acute pharyngitis caused by Candida albicans. Which action is appropriate for the nurse to include in the plan of care? (Exam 1) a. Teach patient to "swish and swallow" prescribed oral nystatin. b. Teach the patient to sleep in a warm, dry environment. c. Avoid giving the patient warm food or warm liquids to drink. d. Assess patient for allergies to penicillin antibiotics.

A. Teach patient to "swish and swallow" prescribed oral nystatin Response Feedback: Oral or pharyngeal fungal infections are treated with nystatin solution. The goal of the "swish and swallow" technique is to expose all the oral mucosa to the antifungal agent. Warm liquids may be soothing to a sore throat. The patient should be taught to use a cool mist humidifier. There is no need to assess for penicillin allergy because C. albicans infection is treated with antifungals.

A patient is receiving a 3% saline continuous IV infusion for hyponatremia. Which assessment data will require the most rapid response by the nurse? (Exam 1) a. There are crackles throughout both lung fields. b. The patient's blood pressure increases to 142/94 mm Hg. c. There is sediment and blood in the patient's urine. d. The patient's radial pulse is 105 beats/min.

A. There are crackles throughout both lung fields Response Feedback: Crackles throughout both lungs suggest that the patient may be experiencing pulmonary edema, a life-threatening adverse effect of hypertonic solutions. The increased pulse rate and blood pressure and the appearance of the urine should also be reported, but they are not as dangerous as the presence of fluid in the alveoli.

A patient who has a history of chronic obstructive pulmonary disease (COPD) was hospitalized for increasing shortness of breath and chronic hypoxemia (SaO2 levels of 89% to 90%). In planning for discharge, which action by the nurse will be most effective in improving compliance with discharge teaching? a. Arrange for the patient's caregiver to be present during the teaching. b. Start giving the patient discharge teaching during the admission process. c. Have the patient repeat the instructions immediately after teaching. d. Accomplish the patient teaching just before the scheduled discharge.

A. arrange for the patient's caregiver to be present during the teaching Response Feedback: Hypoxemia interferes with the patient's ability to learn and retain information, so having the patient's caregiver present will increase the likelihood that discharge instructions will be followed. Having the patient repeat the instructions will indicate that the information is understood at the time, but it does not guarantee retention of the information. Because the patient is likely to be distracted just before discharge, giving discharge instructions just before discharge is not ideal. The patient is likely to be anxious and even more hypoxemic than usual on the day of admission, so teaching about discharge should be postponed.

A young adult patient with cystic fibrosis (CF) is admitted to the hospital with increased dyspnea. Which intervention should the nurse include in the plan of care? (Exam 1) a. Perform chest physiotherapy every 4 hours. b. Place the patient on a low-sodium diet. c. Arrange for a hospice nurse visit. d. Schedule a sweat chloride test.

A. perform chest physiotherapy every 4 hours Response Feedback: Routine scheduling of airway clearance techniques is an essential intervention for patients with CF. A sweat chloride test is used to diagnose CF, but it does not provide any information about the effectiveness of therapy. There is no indication that the patient is terminally ill. Patients with CF lose excessive sodium in their sweat and require high amounts of dietary sodium.

A client admitted to the hospital with a suspected diagnosis of acute pancreatitis is being assessed by the nurse. Which assessment findings would be consistent with acute pancreatitis? Select all that apply (Exam 5)

Abdominal guarding and tenderness. Left upper quadrant pain with radiation to the back. Gray-blue color at the flank.

Prior to administering a client's scheduled dose of enoxaparin sodium, which finding should the nurse evaluate? (Exam 4)

Activate partial prothrombin time (aPTT)

A client has just had a hemorrhoidectomy. Which nursing interventions are appropriate for this client? Select all that apply (Exam 5)

Administer stool softeners as prescribed Encourage a high-fiber diet to promote bowel movements without straining Apply cold packs to the anal-rectal area over the dressing until the packing is removed

An abdominal cholecystectomy is performed on a client with gangrene of the gallbladder. During the first 24 hours postoperatively, when should analgesics be administered? (Exam 5)

As prescribed by a health care provider

The nurse is reviewing the EKG strip rhythm strip. The p waves and qrs complexes are regular. The pr interval is 0.16 seconds

Monitor for any rhythm change

A patient with diabetes who has bacterial pneumonia is being treated with IV gentamicin. What should the nurse monitor for adverse effects of the medication? (Exam 1) a. Blood glucose b. Serum creatinine c. Urine osmolality d. Serum potassium

B. Serum creatinine Response Feedback: When a patient with diabetes, which increases risk for chronic kidney disease (CKD), receives a potentially nephrotoxic medication, it is important to monitor renal function with BUN and creatinine levels. The other laboratory values would not be useful in assessing for the adverse effects of the gentamicin.

A 72-yr-old patient with a history of benign prostatic hyperplasia (BPH) is admitted with acute urinary retention and elevated blood urea nitrogen (BUN) and creatinine levels. Which prescribed therapy should the nurse implement first? (Exam 1) a. Draw a complete blood count. b. Insert urethral catheter. c. Infuse normal saline at 50 mL/hr. d. Obtain renal ultrasound.

B. insert urethral catheter Response Feedback: The patient's elevation in BUN is most likely associated with hydronephrosis caused by the acute urinary retention, so the insertion of an indwelling catheter is the first action to prevent ongoing postrenal failure for this patient. The other actions are appropriate but should be implemented after the catheter.

While assessing the health of a female client, the nurse suspects endocrine dysfunction. Which findings support the nurse's suspicion? Select all that apply (Exam 5)

Blood pressure of 172/80 mm Hg Protrusion of eyeballs Diminished pubic hair

A nurse is caring for a client who had an open abdominal cholecystectomy because of biliary colic. Which nursing action is most important during the postoperative period? (Exam 5)

Encourage coughing and deep breathing

Which client will have the lowest risk for developing hepatotoxicity? Client A - everolimus Client B - Azathioprine Client C - Daclizumab Client D - Cyclosporine (Exam 5)

Client C

The nurse is planning to teach a client with gastroesophageal reflux disease (GERD) about substances to avoid. Which items should the nurse include on this list? Select all that apply (Exam 5)

Chocolate Coffee Fried chicken Peppermint

A client with valvular heart disease who has a clot in the right atrium is receiving a heparin sodium infusion at 1000 units/hr and warfarin sodium 7.5 mg at 5:00 p.m. daily. The morning laboratory results are as follows: activated partial thromboplastin time (aPTT), 32 seconds; international normalized ratio (INR), 1.3. The nurse should take which action based on the client's laboratory results?

Collaborate with the PHCP to obtain a prescription to increase the heparin infusion and continue the warfarin sodium as prescribed.

The nurse is assisting the primary healthcare provider, who is examining the client's skull radiograph. An abnormality in the endocrine gland situated in a depression of the sphenoid bone is suspected. Which hormone release is most probably affected? (Exam 5)

Corticotropin

The nurse identifies a patient with type 1 diabetes and a history of herpes simplex infection as being at risk for Bell's palsy. Which information should the nurse include in teaching the patient? (Exam 3) a."You may be able to prevent Bell's palsy by doing facial exercises regularly." b."Prophylactic treatment of herpes with antiviral agents prevents Bell's palsy." c."Medications to treat Bell's palsy work only if started before paralysis onset." d."Call the doctor if you experience pain or develop herpes lesions near the ear."

D. "Call the doctor if you experience pain or develop herpes lesions near the ear"

The public health nurse is planning a program to decrease the incidence of meningitis in teenagers and young adults. Which action is most likely to be effective? (Exam 3)

Encourage immunization

The nurse notes that a patient who was admitted with heart failure has jugular venous distention (JVD) when lying flat in bed. Which follow-up action should the nurse take next? (Exam 2) a. Obtain vital signs, including oxygen saturation. B. Have the patient perform the Valsalva maneuver. C. Document this JVD finding in the patient's record. D. Observe for JVD with the patient elevated 45 degrees.

D. Observe for JVD with the patient elevated 45 degrees.

Nursing staff on a hospital unit are reviewing rates of health care-associated infections (HAI) of the urianry tract. Which nursing actions will be most helpful in decreasing the risk for urinary HAIR in patients admitted to the hospital? (Exam 1) a. providing perineal hygiene to patients daily and as needed b. testing urine with a dipstick daily for nitrites c. Encouraging adequate oral fluid and nutritional intake d. Avoiding unnecessary urinary catheterization

D. avoiding unnecessary urinary catheterization Because catheterization bypasses many of the protective mechanisms that prevent urinary tract infection (UTI), avoidance of catheterization is the most effective means of reducing HAI. The other actions will also be helpful but are not as useful as decreasing urinary catheter use.

The nurse administers prescribed therapies for a patient with cor pulmonale and right-sided heart failure. Which assessment could be used to evaluate the effectiveness of the therapies? (Exam 1) a. Auscultate for crackles in the lungs. b. Palpate for heaves or thrills over the heart. c. Monitor for elevated white blood cell count. d. Observe for distended neck veins.

D. observe for distended neck veins Response Feedback: Cor pulmonale is right ventricular failure caused by pulmonary hypertension, so clinical manifestations of right ventricular failure such as peripheral edema, jugular venous distention, and right upper-quadrant abdominal tenderness would be expected. Crackles in the lungs are likely to be heard with left-sided heart failure. Findings in cor pulmonale include evidence of right ventricular hypertrophy on electrocardiography and an increase in intensity of the second heart sound. Heaves or thrills are not common with cor pulmonale. White blood count elevation might indicate infection but is not expected with cor pulmonale.

A 12-year-old child with type 2 diabetes is scheduled for abdominal surgery. Which factors are most important for the nurse to consider during the postoperative period? Select all that apply. (Exam 5)

Diabetic control is usually maintained with insulin after surgery The blood glucose level will increase because of stress of surgery

Allopurinol is prescribed for a patient and the nurse provides medication instruction. Which instruction would the nurse provide to the client? (Exam 4)

Drink 3000mL of fluid a day

A client with jaundice reports fatigue, abdominal pain, loss of appetite, dark urine, nausea, and vomiting. Which pathogen is most likely responsible for the client's condition?A client with jaundice reports fatigue, abdominal pain, loss of appetite, dark urine, nausea, and vomiting. Which pathogen is most likely responsible for the client's condition? (Exam 5)

Hepatitis C virus

A nurse is caring for a client with endocrine problems. Which lab finding will alert the nurse that aldosterone will be released? (Exam 5)

Hyponatremia

Four days after abdominal surgery a client has not passed flatus and there are no bowel sounds. Paralytic ileus is suspected. What does the nurse conclude is the most likely cause of the ileus? (Exam 5)

Impaired neural functioning

A nurse is receiving end-of-shift report. Which client should the nurse assess first? (Exam 4)

New onset confusion in a client who has a left femur fracture

The nurse teaches the client about endocrine functioning of Part A. Which information from the client indicates successful learning? (Exam 5)

Part A is the pituitary gland - promotes growth

A nurse in the postanesthesia care unit (PACU) is providing care to a client who had an abdominal cholecystectomy and observes serosanguineous drainage on the abdominal dressing. What is the next nursing action? (Exam 5)

Reinforce the dressing

A nurse provides teaching to a client who has a newly applied short arm fiberglass cast. Which of the following instructions should the nurse discuss? (Exam 4)

Report numbness or tingling to your provider Keep arm elevated above the heart during rest Blow air from hair dryer to relieve itching Wrap cast with plastic covering prior to showering

A client with a family history of diabetes is concerned about the effects of psychiatric medication on the endocrine system. Which psychotropic medication is most likely to cause metabolic syndrome? (Exam 5)

Risperidone

The primary healthcare provider prescribed a diagnostic study with contrast medium for an older adult who has an endocrine disorder. Which assessment result should the nurse check before the study? (Exam 5)

Serum creatinine

The client has developed a fib with a ventricular rate of 150 beats per minute. Which associated findings would the nurse anticipate in the assessment? SATA

Syncope Dizziness Palpitations

A patient arrived at the ED after tripping over a rug and falling at home. Which finding should the nurse identify as most important to communicate to the HCP? (Exam 4)

The right arm appears to be shorter than the left

The nurse is monitoring a client with a diagnosis of peptic ulcer. Which assessment finding would most likely indicate perforation of the ulcer? (Exam 5)

a rigid, board-like abdomen

Which patient is most appropriate for the intensive care unit (ICU) charge nurse to assign to a registered nurse (RN) who has floated from the medical unit? (Exam 3) a. A 45-year-old receiving IV antibiotics for meningococcal meningitis b. A 25-year-old admitted with a skull fracture and craniotomy the previous day c. A 55-year-old who has increased intracranial pressure (ICP) and is receiving hyperventilation therapy d. A 35-year-old with ICP monitoring after a head injury last week

a. A 45-year-old receiving IV antibiotics for meningococcal meningitis

What finding should indicate to the nurse that colchicine has been effective for a patient with an acute attack of gout? (Exam 4) a. Reduced joint pain b. Increased urine output c. Elevated serum uric acid d. Increased white blood cells

a. Reduced joint pain

Which patient should the nurse assign as the roommate for a patient who has aplastic anemia? (Exam 6) a. A patient with chronic heart failure b. A patient who has viral pneumonia c. A patient who has right leg cellulitis d. A patient with multiple abdominal drains

a. A patient with chronic heart failure

The nurse is caring for a young adult client who had a mandatory electrocardiogram (ECG) before participating on a college swim team and is found to have sinus bradycardia, rate 52. BP is 114/54 mm Hg, and the student denies any health problems. Which of the following actions by the nurse is best? (exam 2) a. Allow the student to participate on the swim team. b. Refer the student to a cardiologist for further assessment. c. Obtain more detailed information about the student's health history. d. Tell the student to stop swimming immediately if any dyspnea occurs.

a. Allow the student to participate on the swim team.

A 40-year-old patient has a ruptured cerebral aneurysm and subarachnoid hemorrhage. Which intervention will be included in the care plan? (Exam 3) a. Apply intermittent pneumatic compression stockings. b. Assist to dangle on edge of bed and assess for dizziness. c. Encourage patient to cough and deep breathe every 4 hours. d. Insert an oropharyngeal airway to prevent airway obstruction.

a. Apply intermittent pneumatic compression stockings.

The nurse admits a terminally ill patient to the hospital. What is the first action that the nurse should complete when planning this patients care? (Exam 6) a. Determine the patients wishes regarding end-of-life care. b. Emphasize the importance of addressing any family issues. c. Discuss the normal grief process with the patient and family. d. Encourage the patient to talk about any fears or unresolved issues.

a. Determine the patients wishes regarding end-of-life care.

As the nurse admits a patient in end-stage kidney disease to the hospital, the patient tells the nurse, If my heart or breathing stop, I do not want to be resuscitated. Which action is best for the nurse to take? (Exam 6) a. Ask if these wishes have been discussed with the health care provider. b. Place a Do Not Resuscitate (DNR) notation in the patients care plan. c. Inform the patient that a notarized advance directive must be included in the record or resuscitation must be performed. d. Advise the patient to designate a person to make health care decisions when the patient is not able to make them independently.

a. Ask if these wishes have been discussed with the health care provider.

After placement of a centrally inserted IV catheter, a patient reports acute chest pain and dyspnea. Which action should the nurse take first? (Exam 1) a. Auscultate the patient's breath sounds. b. Notify the health care provider. c. Offer reassurance to the patient. d. Give prescribed PRN morphine sulfate IV.

a. Auscultate the patient's breath sounds Response Feedback: The initial action should be to assess the patient further because the history and symptoms are consistent with several possible complications of central line insertion, including embolism and pneumothorax. The other actions may be appropriate, but further assessment of the patient is needed before notifying the health care provider, offering reassurance, or administration of morphine.

Discharge teaching for the patient who underwent laparoscopic cholecystectomy should include the need to (Exam 5)

call is there are any changes in stool or urine color

Which assessment finding for an older patient indicates that the nurse should take immediate action? (Exam 1) a. Bilateral basilar crackles b. Dry mucous membranes c. Barrel-shaped chest d. Weak cough effort

a. Bilateral basilar crackles Response Feedback: Crackles in the lower half of the lungs indicate that the patient may have an acute problem such as heart failure. The nurse should immediately accomplish further assessments, such as O2 saturation, and notify the health care provider. A barrel-shaped chest, and a weak cough effort are associated with aging and immediate action is not indicated. An older patient has a less forceful cough and fewer and less functional cilia. Mucous membranes tend to be drier.

A patient who comes to the clinic reports frequent, watery stools for 2 days. Which action should the nurse take first? (Exam 1) a. Check the patient's blood pressure. b. Ask about extremity numbness or tingling. c. Draw blood for serum electrolyte levels. d. Obtain the baseline weight.

a. Check the patient's blood pressure. Response Feedback: Because the patient's history suggests that fluid volume deficit may be a problem, assessment for adequate circulation is the highest priority. The other actions are also appropriate but are not as essential as determining the patient's perfusion status.

Which finding from analysis of fluid from a patient's right knee arthrocentesis should be of concern to the nurse? (Exam 4) a. Cloudy fluid b. Scant thin fluid c. Pale yellow fluid d. Straw-colored fluid

a. Cloudy fluid

A nurse assesses a postoperative patient 2 days after chest surgery. What findings indicate that the patient requires better pain management (select all that apply)? (Exam 6) a. Confusion d. Shallow breathing b. Hypoglycemia e. Elevated temperature c. Poor cough effort

a. Confusion d. Shallow breathing e. Elevated temperature c. Poor cough effort

The nurse plans a presentation for community members about how to decrease the risk for antibiotic- resistant infections. Which information will the nurse include in the teaching plan (select all that apply)? (Exam 6) a. Continue taking antibiotics until all the medication is gone. b. Antibiotics may sometimes be prescribed to prevent infection. c. Unused antibiotics that are more than a year old should be discarded. d. Antibiotics are effective in treating influenza associated with high fevers. e. Hand washing is effective in preventing many viral and bacterial infections.

a. Continue taking antibiotics until all the medication is gone. b. Antibiotics may sometimes be prescribed to prevent infection. e. Hand washing is effective in preventing many viral and bacterial infections.

A patient develops neutropenia after receiving chemotherapy. Which information about ways to prevent infection will the nurse include in the teaching plan (select all that apply)? (Exam 6) a. Cook food thoroughly before eating. b. Choose low fiber, low residue foods. c. Avoid public transportation such as buses. d. Use rectal suppositories if needed for constipation. e. Talk to the oncologist before having any dental work done.

a. Cook food thoroughly before eating. c. Avoid public transportation such as buses. e. Talk to the oncologist before having any dental work done.

A patient who has bacterial meningitis is disoriented and anxious. Which nursing action will be included in the plan of care? (Exam 3) a. Encourage family members to remain at the bedside. b. Apply soft restraints to protect the patient from injury. c. Keep the room well-lighted to improve patient orientation. d. Minimize contact with the patient to decrease sensory input.

a. Encourage family members to remain at the bedside.

The nurse cares for a terminally ill patient who is experiencing pain that is continuous and severe. How should the nurse schedule the administration of opioid pain medications? (Exam 6) a. Give around-the-clock routine administration of analgesics. b. Provide PRN doses of medication whenever the patient requests. c. Offer enough pain medication to keep the patient sedated and unaware of stimuli. d. Suggest analgesic doses that provide pain control without decreasing respiratory rate.

a. Give around-the-clock routine administration of analgesics.

The nurse cares for a patient infected with human immunodeficiency virus (HIV) who has just been diagnosed with asymptomatic chronic HIV infection. Which prophylactic measures will the nurse include in the plan of care (select all that apply)? (Exam 6) a. Hepatitis B vaccine b. Pneumococcal vaccine c. Influenza virus vaccine d. Trimethoprim-sulfamethoxazole e. V aricella zoster immune globulin

a. Hepatitis B vaccine b. Pneumococcal vaccine c. Influenza virus vaccine

A patient has been taking phenytoin (Dilantin) for 2 years. Which action will the nurse take when evaluating for adverse effects of the medication? (Exam 3) a. Inspect the oral mucosa. b. Listen to the lung sounds. c. Auscultate the bowel tones. d. Check pupil reaction to light.

a. Inspect the oral mucosa.

A patient receives 3% NaCl solution for correction of hyponatremia. Which assessment is most important for the nurse to monitor while the patient is receiving this infusion? (Exam 1) a. Lung sounds b. Peripheral pulses c. Urinary output d. Peripheral edema

a. Lung sounds Response Feedback: Hypertonic solutions cause water retention, so the patient should be monitored for symptoms of fluid excess. Crackles in the lungs may indicate the onset of pulmonary edema and are a serious manifestation of fluid excess. Peripheral pulses, peripheral edema, or changes in urine output are also important to monitor when administering hypertonic solutions, but they do not indicate acute respiratory or cardiac decompensation.

Which actions should the nurse include in the plan of care for a patient with metastatic bone cancer of the left femur? (Exam 4) a. Monitor serum calcium b. Teach about the need for strict bed rest c. Explain the use of sustained-release opioids d. Support the left leg when repositioning the patient e. Assist family and patient as they discuss the prognosis

a. Monitor serum calcium c. Explain the use of sustained-release opioids d. Support the left leg when repositioning the patient e. Assist family and patient as they discuss the prognosis

A patient with an open leg leison has a white blood cell (WBC) count of 13, 500/L and a band count of 11%. What action should the nurse take first? (Exam 6) a. Obtain wound cultures. b. Start antibiotic therapy. c. Redress the wound with wet-to-dry dressings. d. Continue to monitor the wound for purulent drainage

a. Obtain wound cultures.

A patient with chronic obstructive pulmonary disease (COPD) has been eating very little and has lost weight. Which intervention would be most appropriate for the nurse to include in the plan of care? (Exam 1) a. Offer high-calorie protein snacks between meals and at bedtime. b. Encourage increased intake of whole grains. c. Increase the patient's menu order of fruits and fruit juices. d. Assist the patient in choosing foods with high vegetable content.

a. Offer high-calorie protein snacks between meals and at bedtime. Response Feedback: Eating small amounts more often (as occurs with snacking) will increase caloric intake by decreasing the fatigue and feelings of fullness associated with large meals. Patients with COPD should rest before meals. Foods that have a lot of texture such as whole grains may take more energy to eat and get absorbed and lead to decreased intake. Although fruits, juices, and minerals are not contraindicated, foods high in protein are a better choice.

Which menu choice indicates that the patient understands the nurses teaching about best dietary choices for iron-deficiency anemia? (Exam 6) a. Omelet and whole wheat toast b. Cantaloupe and cottage cheese c. Strawberry and banana fruit plate d. Cornmeal muffin and orange juice

a. Omelet and whole wheat toast

The nurse at the clinic is interviewing a 64-year-old woman who is 5 feet, 3 inches tall and weighs 125 pounds (57 kg). The patient has not seen a health care provider for 20 years. She walks 5 miles most days and has a glass of wine 2 or 3 times a week. Which topics will the nurse plan to include in patient teaching about cancer screening and decreasing cancer risk (select all that apply)? (Exam 6) a. Pap testing b. Tobacco use c. Sunscreen use d. Mammography e. Colorectal screening

a. Pap testing c. Sunscreen use d. Mammography e. Colorectal screening

A patient admitted with acute kidney injury due to dehydration has oliguria, anemia, and hyperkalemia. Which prescribed action should the nurse take first? (Exam 1) a. Place the patient on a cardiac monitor. b. Give sodium polystyrene sulfonate (Kayexalate). c. Insert a urinary retention catheter. d. Administer epoetin alfa (Epogen).

a. Place the patient on a cardiac monitor. Response Feedback: Because hyperkalemia can cause fatal dysrhythmias, the initial action should be to monitor the cardiac rhythm. Kayexalate and Epogen will take time to correct the hyperkalemia and anemia. The catheter allows monitoring of the urine output but does not correct the cause of the renal failure.

Which nursing actions for the care of a dying patient can the nurse delegate to a licensed practical/vocational nurse (LPN/LVN) (select all that apply)? (Exam 6) a. Provide postmortem care to the patient. b. Encourage the family members to talk with and reassure the patient. c. Determine how frequently physical assessments are needed for the patient. d. Teach family members about commonly occurring signs of approaching death. e. Administer the prescribed morphine sulfate sublingual as necessary for pain control.

a. Provide postmortem care to the patient. b. Encourage the family members to talk with and reassure the patient. e. Administer the prescribed morphine sulfate sublingual as necessary for pain control.

3. A patient with a family history of cystic fibrosis (CF) asks for information about genetic testing. Which response by the nurse is most appropriate? (Exam 6) a. Refer the patient to a qualified genetic counselor. b. Ask the patient why genetic testing is so important. c. Remind the patient that genetic testing has many social implications. d. Tell the patient that cystic fibrosis is an autosomal recessive disor

a. Refer the patient to a qualified genetic counselor.

A 25-yr-old patient has been admitted with a severe crushing injury after an industrial accident. Which laboratory result will be most important to report to the health care provider? (Exam 1) a. Serum potassium level of 6.5 mEq/L b. Blood urea nitrogen (BUN) of 56 mg/dL c. White blood cell count of 11,500/µL d. Serum creatinine level of 2.1 mg/dL

a. Serum potassium level of 6.5 mEq/L Response Feedback: The hyperkalemia associated with crushing injuries may cause cardiac arrest and should be treated immediately. The nurse will report the other laboratory values, but abnormalities in these are not immediately life threatening.

A hospitalized 31-year-old patient with a history of cluster headache awakens during the night with a severe stabbing headache. Which action should the nurse take first? (Exam 3) a. Start the ordered PRN oxygen at 6 L/min. b. Put a moist hot pack on the patient's neck. c. Give the ordered PRN acetaminophen (Tylenol). d. Notify the patient's health care provider immediately.

a. Start the ordered PRN oxygen at 6 L/min.

A patient in the outpatient clinic has a new diagnosis of peripheral artery disease (PAD). Which group of drugs will the nurse plan to include when teaching about PAD management? (Exam 2) a. Statins b. Antibiotics c. Thrombolytics d. Anticoagulants

a. Statins

The nurse prepares a patient who has a left-sided pleural effusion for a thoracentesis. How should the nurse position the patient? (Exam 1) a. Sitting upright with the arms supported on an over bed table b. High-Fowler's position with the left arm extended c. On the right side with the left arm extended above the head d. Supine with the head of the bed elevated 30 degrees

a. Supine upright with the arms supported on an over bed table Response Feedback: The upright position with the arms supported increases lung expansion, allows fluid to collect at the lung bases, and expands the intercostal space so that access to the pleural space is easier. The other positions would increase the work of breathing for the patient and make it more difficult for the health care provider performing the thoracentesis.

Which action should the nurse take when caring for a patient with osteomalacia? (Exam 4) a. Teach about the use of vitamin D supplements b. Educate about the need for weight-bearing exercise c. Instruct the patient to avoid dairy products in the diet d. Discuss the use of medications such as bisphosphonates

a. Teach about the use of vitamin D supplements

A female patient with a suspected urinary tract infection is to provide a clean-catch urine specimen for culture and sensitivity testing. What should the nurse do to obtain the specimen? (Exam 1) a. Tell the patient to clean the urethral area, void a small amount into the toilet, then void directly into a sterile container. b. Clean the area around the patient's meatus with a povidone iodine (Betadine) swab and then have the patient void into a sterile specimen cup. c. Insert a short sterile "mini" catheter attached to a collecting container into the urethra and bladder to obtain the specimen. d. Have the patient empty the bladder completely; then obtain the next urine specimen that the patient is able to void.

a. Tell the patient to clean the urethral area, void a small amount into the toilet, then void directly into a sterile container. Response Feedback: This answer describes the technique for obtaining a clean-catch specimen. The answer beginning, "insert a short, small, 'mini' catheter attached to a collecting container" describes a technique that would result in a sterile specimen, but a health care provider's order for a catheterized specimen would be required. Using Betadine before obtaining the specimen might result in suppressing the growth of some bacteria. The technique described in the answer beginning "have the patient empty the bladder completely" would not result in a sterile specimen.

A patient who is 2 days post femoral popliteal bypass graft to the right leg is being cared for on the vascular unit. Which action by a licensed practical/vocational nurse (LPN/LVN) caring for the patient requires the registered nurse (RN) to intervene? (Exam 2) a. The LPN/LVN has the patient to sit in a chair for 2 hours. b. The LPN/LVN gives the prescribed aspirin after breakfast. c. The LPN/LVN assists the patient to walk 40 feet in the hallway. d. The LPN/LVN places the patient in Fowler's position for meals.

a. The LPN/LVN has the patient to sit in a chair for 2 hours.

A patient is being evaluated for a possible spinal cord tumor. Which findings should the nurse recognize as needing the most immediate action? (Exam 3) a. The patient has new onset weakness of both legs. b. The patient complains of chronic severe back pain. c. The patient starts to cry and says, "I feel hopeless." d. The patient expresses anxiety about having surgery.

a. The patient has new onset weakness of both legs.

Which question asked by the nurse will give the most information about the patients metastatic bone cancer pain? (Exam 6) a. How long have you had this pain? b. How would you describe your pain? c. How much medication do you take for the pain? d. How many times a day do you take medication for the pain?

b. How would you describe your pain?

When admitting a 42-year-old patient with a possible brain injury after a car accident to the emergency department (ED), the nurse obtains the following information. Which finding is most important to report to the health care provider? (Exam 3) a. The patient takes warfarin (Coumadin) daily. b. The patient's blood pressure is 162/94 mm Hg. c. The patient is unable to remember the accident. d. The patient complains of a severe dull headache.

a. The patient takes warfarin (Coumadin) daily.

When caring for a patient who experienced a T2 spinal cord transection 24 hours ago, which collaborative and nursing actions will the nurse include in the plan of care (select all that apply)? (Exam 3) a. Urinary catheter care b. Nasogastric (NG) tube feeding c. Continuous cardiac monitoring d. Maintain a warm room temperature e. Administration of H2 receptor blockers

a. Urinary catheter care c. Continuous cardiac monitoring d. Maintain a warm room temperature e. Administration of H2 receptor blockers

The nurse is titrating the IV fluid infusion rate immediately after a patient has had kidney transplantation. Which parameter will be most important for the nurse to consider? (Exam 1) a. Urine output b. Blood urea nitrogen (BUN) level c. Creatinine clearance d. Heart rate

a. Urine output Response Feedback: Fluid volume is replaced based on urine output after transplant because the urine output can be as high as a liter an hour. The other data will be monitored but are not the most important determinants of fluid infusion rate.

The nurse is caring for a client with biliary cancer. The associated jaundice gets progressively worse. The nurse is most concerned about the potential complication of what symptom? (Exam 5)

bleeding

A patient is admitted to the hospital with possible acute pericarditis. The nurse should plan to teach the patient about the purpose of: (Exam 2) a. echocardiography. b. daily blood cultures. c. cardiac catheterization. d. 24-hour Holter monitor.

a. echocardiography.

A patient who was involved in a motor vehicle crash has had a tracheostomy placed to allow for continued mechanical ventilation. How should the nurse interpret the following arterial blood gas results: pH 7.48, PaO2 85 mm Hg, PaCO2 32 mm Hg, and HCO3 25 mEq/L? (Exam 1) a. Respiratory alkalosis b. Metabolic alkalosis c. Metabolic acidosis d. Respiratory acidosis

a. respiratory alkalosis The pH indicates that the patient has alkalosis and the low PaCO2 indicates a respiratory cause. The other responses are incorrect based on the pH and the normal HCO3.

The clinic nurse is teaching a patient with acute sinusitis. Which interventions should the nurse plan to include in the teaching session? (Select all that apply.) (Exam 1) a. Taking a hot shower will increase sinus drainage and decrease pain. b. Decongestants can be used to relieve swelling. c. Avoid blowing the nose to decrease the nosebleed risk. d. Saline nasal spray can be made at home and used to wash out secretions. e. You will be more comfortable if you keep your head in an upright position.

a. taking a hot shower will increase sinus drainage and decrease pain b. decongestants can be used to relieve swelling d. saline nasal spray can be made at home and used to wash out secretions e. You will be more comfortable if you keep your head in an upright position Response Feedback: The steam and heat from a shower will help thin secretions and improve drainage. Decongestants can be used to relieve swelling. Patients can use either over-the-counter sterile saline solutions or home-prepared saline solutions to thin and remove secretions. Maintaining an upright posture decreases sinus pressure and the resulting pain. Blowing the nose after a hot shower or using the saline spray is recommended to expel secretions.

A patient with cancer that has metastasized to the liver has symptoms of fluid retention, including edema and ascites. To determine the effectiveness of diuretic therapy on ascites, what should the nurse assess? (Exam 5)

abdominal girth

To assess for functional deficits, which question will the nurse ask a patient who has been admitted for treatment of a benign occipital lobe tumor? (Exam 3) a. "Do you have difficulty in hearing?" b. "Are you experiencing visual problems?" c. "Are you having any trouble with your balance?" d. "Have you developed any weakness on one side?"

b. "Are you experiencing visual problems?"

The home health nurse cares for an alert and oriented older adult patient with a history of dehydration. Which instructions should the nurse give this patient related to fluid intake? (Exam 1) a. "Drink more fluids in the late evening." b. "Increase the fluids if your mouth feels dry." c. "If you feel confused, you need more fluids." d. "More fluids are needed if you feel thirsty."

b. "Increase the fluids if your mouth feels dry." Response Feedback: An alert older patient will be able to self-assess for signs of oral dryness such as thick oral secretions or dry-appearing mucosa. The thirst mechanism decreases with age and is not an accurate indicator of volume depletion. Many older patients prefer to restrict fluids slightly in the evening to improve sleep quality. The patient will not be likely to notice and act appropriately when changes in level of consciousness occur.

The nurse teaches a patient about the transmission of pulmonary tuberculosis (TB). Which statement, if made by the patient, indicates that teaching was effective? (Exam 1) a. "I will keep the windows closed at home." b. "My spouse will sleep in another room." c. "I will stay indoors whenever possible." d. "I will take the bus instead of driving."

b. "my spouse will sleep in another room." Response Feedback: Teach the patient how to minimize exposure to close contacts and household members. Homes should be well ventilated, especially the areas where the infected person spends a lot of time. While still infectious, the patient should sleep alone, spend as much time as possible outdoors, and minimize time in congregate settings or on public transportation.

Which information should the nurse include when teaching a patient with newly diagnoses systemic exertion intolerance disease (SEID) about self-management (Exam 4) a. Symptoms usually progress as patients become older b. A gradual increase in daily exercise may help decrease fatigue c. Avoid use of over-the-counter antihistamines or decongestants d. A low-residue, low-fiber diet will reduce any abdominal distention

b. A gradual increase in daily exercise may help decrease fatigue

The nurse is caring for a patient who is recovering from a sudden cardiac death (SCD) event and has no evidence of an acute myocardial infarction (AMI). What should the nurse anticipate teaching the patient? (Exam 2) a. Sudden cardiac death events rarely reoccur. b. Additional diagnostic testing will be required. c. Long-term anticoagulation therapy will be needed. d. Limiting physical activity will prevent future SCD events.

b. Additional diagnostic testing will be required

A patient with terminal cancer-related pain and a history of opioid abuse complains of breakthrough pain 2 hours before the next dose of sustained-release morphine sulfate (MS Contin) is due. Which action should the nurse take first? (Exam 6) a. Use distraction by talking about things the patient enjoys. b. Administer the prescribed PRN immediate-acting morphine. c. Suggest the use of alternative therapies such as heat or cold. d. Consult with the doctor about increasing the MS Contin dose.

b. Administer the prescribed PRN immediate-acting morphine.

The nurse is evaluating a client's response to cardioversion. Which observation would be of highest priority to the nurse? (Exam 2) a. Blood pressure b. Airway patency c. Oxygen flow rate d. Level of consciousness

b. Airway patency

Which information will the nurse include when preparing teaching materials for a patient with an exacerbation of rheumatoid arthritis? (Exam 4) a. Affected joints should not be exercised when pain is present. b. Applying cold packs before exercise may decrease joint pain. c. Exercises should be performed passively by someone other than the patient. d. Walking may substitute for range-of-motion (ROM) exercises on some days

b. Applying cold packs before exercise may decrease joint pain.

A patient tells the nurse, I would like to use a home genetic test to see if I will develop breast cancer. Which response by the nurse is best? (Exam 6) a. Home genetic testing is very expensive. b. Are you concerned about developing breast cancer? c. Wont you be depressed if the testing shows a positive result? d. Genetic testing can only determine if you are at higher risk for breast cancer.

b. Are you concerned about developing breast cancer?

A patient who is anxious and has difficulty breathing seeks treatment after being stung by a wasp. What is the nurses priority action? (Exam 6) a. Have the patient lie down. b. Assess the patients airway. c. Administer high-flow oxygen. d. Remove the stinger from the site.

b. Assess the patients airway.

Which nursing action has the highest priority for a patient who was admitted 16 hours previously with a C5 spinal cord injury? (Exam 3) a. Cardiac monitoring for bradycardia b. Assessment of respiratory rate and effort c. Application of pneumatic compression devices to legs d. Administration of methylprednisolone (Solu-Medrol) infusion

b. Assessment of respiratory rate and effort

Several weeks after a stroke, a 50-year-old male patient has impaired awareness of bladder fullness, resulting in urinary incontinence. Which nursing intervention will be best to include in the initial plan for an effective bladder training program? (Exam 3) a. Limit fluid intake to 1200 mL daily to reduce urine volume. b. Assist the patient onto the bedside commode every 2 hours. c. Perform intermittent catheterization after each voiding to check for residual urine. d. Use an external "condom" catheter to protect the skin and prevent embarrassment

b. Assist the patient onto the bedside commode every 2 hours.

A patient with pancytopenia of unknown origin is scheduled for the following diagnostic tests. The nurse will provide a consent form to sign for which test? (Exam 6) a. ABO blood typing b. Bone marrow biopsy c. Abdominal ultrasound d. Complete blood count (CBC)

b. Bone marrow biopsy

A patient has arrived for a scheduled hemodialysis session. Which nursing action is appropriate for the registered nurse (RN) to delegate to a dialysis technician? (Exam 1) a. Assess for causes of an increase in predialysis weight. b. Check blood pressure before starting dialysis. c. Teach the patient about fluid restrictions. d. Determine the ultrafiltration rate for the hemodialysis.

b. Check blood pressure before starting dialysis. Response Feedback: Dialysis technicians are educated in monitoring for blood pressure. Assessment, adjustment of the appropriate ultrafiltration rate, and patient teaching require the education and scope of practice of an RN.

A 20-year-old male patient is admitted with a head injury after a collision while playing football. After noting that the patient has developed clear nasal drainage, which action should the nurse take? (Exam 3) a. Have the patient gently blow the nose. b. Check the drainage for glucose content. c. Teach the patient that rhinorrhea is expected after a head injury. d. Obtain a specimen of the fluid to send for culture and sensitivity

b. Check the drainage for glucose content.

A patient with second-degree burns has been receiving hydromorphone through patient-controlled analgesia (PCA) for a week. The patient wakes up frequently during the night complaining of pain. What action by the nurse is most appropriate? (Exam 6) a. Administer a dose of morphine every 1 to 2 hours from the PCA machine while the patient is sleeping. b. Consult with the health care provider about using a different treatment protocol to control the patients pain. c. Request that the health care provider order a bolus dose of morphine to be given when the patient awakens with pain. d. Teach the patient to push the button every 10 minutes for an hour before going to sleep, even if the pain is minimal.

b. Consult with the health care provider about using a different treatment protocol to control the patients pain.

Which nursing action could the nurse delegate to unlicensed assistive personnel (UAP) when caring for a patient who is using a fentanyl (Duragesic) patch and a heating pad for treatment of chronic back pain? (Exam 6) a. Check the skin under the heating pad. b. Count the respiratory rate every 2 hours. c. Monitor sedation using the sedation assessment scale. d. Ask the patient about whether pain control is effective.

b. Count the respiratory rate every 2 hours.

A hospice nurse who has become close to a terminally ill patient is present in the home when the patient dies and feels saddened and tearful as the family members begin to cry. Which action should the nurse take at this time? (Exam 6) a. Contact a grief counselor as soon as possible. b. Cry along with the patients family members. c. Leave the home as soon as possible to allow the family to grieve privately. d. Consider whether working in hospice is desirable because patient losses are common.

b. Cry along with the patients family members.

Which action for the care of a patient who has scleroderma can the registered nurse (RN) delegate to unlicensed assistive personnel? (Exam 4) a. Monitor for difficulty in breathing b. Document the patient's oral intake c. Check finger strength and movement d. Apply capsaicin (Zostrix) cream to the hands

b. Document the patient's oral intake

Which action should the nurse include in the plan of care for a patient with a new diagnosis of rheumatoid arthritis? (Exam 4) a. Instruct the patient to purchase a soft mattress b. Encourage the patient to take a nap in the afternoon c. Teach the patient to use lukewarm water when bathing d. Suggest exercise with light weights several times daily

b. Encourage the patient to take a nap in the afternoon

Which action will the public health nurse take to reduce the incidence of epidemic encephalitis in a community? (Exam 3) a. Teach about prophylactic antibiotics after exposure to encephalitis. b. Encourage the use of effective insect repellent during mosquito season. c. Remind patients that most cases of viral encephalitis can be cared for at home. d. Arrange to screen school-age children for West Nile virus during the school year.

b. Encourage the use of effective insect repellent during mosquito season.

According to the Center for Disease Control (CDC) guidelines, which personal protective equipment will the nurse put on when assessing a patient who is on contact precautions for diarrhea caused by Clostridium difficile(select all that apply)? (Exam 6) a. Mask b. Gown c. Gloves d. Shoe covers e. Eye protection

b. Gown c. Gloves

A patient in the clinic with cystic fibrosis (CF) reports increased sweating and weakness during the summer months. Which action by the nurse would be most appropriate? (Exam 1) a. Teach the patient signs of hypoglycemia. b. Have the patient add dietary salt to meals. c. Suggest decreasing intake of dietary fat and calories. d. Teach the patient about pancreatic enzyme replacement.

b. Have the patient add dietary salt to meals. Response Feedback: Added dietary salt is indicated whenever sweating is excessive, such as during hot weather, when fever is present, or from intense physical activity. The management of pancreatic insufficiency includes pancreatic enzyme replacement before each meal and snack. This patient is at risk for hyponatremia based on reported symptoms. Adequate intake of fat, calories, protein, and vitamins is important. Fat-soluble vitamins (vitamins A, D, E, and K) must be supplemented because they are malabsorbed. Use of caloric supplements improves nutritional status. Hyperglycemia caused by pancreatic insufficiency is more likely to occur than hypoglycemia.

Which action will the nurse in the hypertension clinic take to obtain an accurate baseline blood pressure (BP) for a new patient? (Exam 2) a. Deflate the BP cuff at a rate of 5 to 10 mm Hg per second. b. Have the patient sit in a chair with the feet flat on the floor. c. Assist the patient to the supine position for BP measurements. d. Obtain two BP readings in the dominant arm and average the results.

b. Have the patient sit in a chair with the feet flat on the floor.

Which nursing action will be most effective in ensuring daily medication compliance for a patient with mild dementia? (Exam 3) a. Setting the medications up monthly in a medication box b. Having the patient's family member administer the medication c. Posting reminders to take the medications in the patient's house d. Calling the patient weekly with a reminder to take the medication

b. Having the patient's family member administer the medication

The nurse is reviewing laboratory results and notes an aPTT level of 28 seconds. The nurse should notify the health care provider in anticipation of adjusting which medication? (Exam 6) a. Aspirin b. Heparin c. Warfarin d. Erythropoietin

b. Heparin

Which nursing action should be included in the plan of care after endovascular repair of an abdominal aortic aneurysm? (Exam 2) a. Record hourly chest tube drainage. b. Monitor fluid intake and urine output. c. Check the abdominal incision for any redness. d. Teach the reason for a prolonged recovery period.

b. Monitor fluid intake and urine output.

The health care provider orders a patient-controlled analgesia (PCA) machine to provide pain relief for a patient with acute surgical pain who has never received opioids in the past. Which nursing actions regarding opioid administration are appropriate at this time (select all that apply)? (Exam 6) a. Assess for signs that the patient is becoming addicted to the opioid. b. Monitor for therapeutic and adverse effects of opioid administration. c. Emphasize that the risk of some opioid side effects increases over time. d. Teach the patient about how analgesics improve postoperative activity levels. e. Provide instructions on decreasing opioid doses by the second postoperative day.

b. Monitor for therapeutic and adverse effects of opioid administration. d. Teach the patient about how analgesics improve postoperative activity levels.

The nurse obtains the following info newly diagnosed with elevated blood pressure. Which finding is most important to address with the patient? (Exam 2) a. Low dietary fiber intake b. No regular physical exercise c. Drinks a beer with dinner every night d. Weight is 5 pounds above ideal weight

b. No regular physical exercise

A patient who is experiencing an asthma attack develops bradycardia and a decrease in wheezing. Which action should the nurse take first? (Exam 1) a. Encourage the patient to cough and deep breathe. b. Notify the health care provider. c. Administer IV methylprednisolone (Solu-Medrol). d. Document changes in respiratory status.

b. Notify the health care provider. Response Feedback: The patient's assessment indicates impending respiratory failure, and the nurse should prepare to assist with intubation and mechanical ventilation after notifying the health care provider. IV corticosteroids require several hours before having any effect on respiratory status. The patient will not be able to cough or deep breathe effectively. Documentation is not a priority at this time.

The nurse is admitting a patient newly diagnosed with peripheral artery disease. Which admission order should the nurse question? (Exam 2) a. Cilostazol drug therapy b. Omeprazole drug therapy c. Use of treadmill for exercise d. Exercise to the point of discomfort

b. Omeprazole drug therapy

Which information about a patient with a lumbar vertebral compression fracture should the nurse immediately report to the HCP? (Exam 4) a. Patient declines to be turned due to back pain b. Patient has been incontinent of urine and stool c. Patient reports lumbar area tenderness to palpation d. Patient frequently uses oral corticosteroids to treat asthma

b. Patient has been incontinent of urine and stool

The nurse who works in the vascular clinic has several patients with venous insufficiency scheduled today. Which patient should the nurse assign to an experienced licensed practical/vocational nurse (LPN/LVN)? (Exam 2) a. Patient who has been complaining of increased edema and skin changes in the legs b. Patient who needs wound care for a chronic venous stasis ulcer on the right lower leg c. Patient who has a history of venous thromboembolism and is complaining of dyspnea d. Patient who needs teaching about elastic compression stockings for venous insufficiency

b. Patient who needs wound care for a chronic venous stasis ulcer on the right lower leg

Which action should the nurse take to prepare a patient with a pleural effusion for a thoracentesis? (Exam 1) a. Remind the patient not to eat or drink 6 hours. b. Position the patient sitting up on the side of the bed. c. Start a peripheral IV line to administer sedation. d. Obtain a collection device to hold 3 liters of pleural fluid.

b. Position the patient sitting up on the side of the bed. Response Feedback: When the patient is sitting up, fluid accumulates in the pleural space at the lung bases and can more easily be located and removed. The patient does not usually require sedation for the procedure, and there are no restrictions on oral intake because the patient is not sedated or unconscious. Usually only 1000 to 1200 mL of pleural fluid is removed at one time. Rapid removal of a large volume can result in hypotension, hypoxemia, or pulmonary edema.

Which assessment finding should the nurse expect when a patient with acute kidney injury (AKI) has an arterial blood pH of 7.30? (Exam 1) a. Persistent skin tenting b. Rapid, deep respirations c. Hot, flushed face and neck d. Bounding peripheral pulses

b. Rapid, deep respirations Response Feedback: Patients with metabolic acidosis caused by AKI may have Kussmaul respirations to eliminate carbon dioxide. Bounding pulses and vasodilation are not associated with metabolic acidosis. Because the patient is likely to have fluid retention, poor skin turgor would not be a finding in AKI.

Which intervention will the nurse include in the plan of care for a patient with moderate dementia who had an appendectomy 2 days ago? (Exam 3) a. Provide complete personal hygiene care for the patient. b. Remind the patient frequently about being in the hospital. c. Reposition the patient frequently to avoid skin breakdown. d. Place suction at the bedside to decrease the risk for aspiration

b. Remind the patient frequently about being in the hospital

. The nurse provides discharge instructions to a patient who has an immune deficiency involving the T lymphocytes. Which screening should the nurse include in the teaching plan for this patient? (Exam 6) a. Screening for allergies b. Screening for cancers c. Antibody deficiency screening d. Screening for autoimmune disorders

b. Screening for cancers

The registered nurse (RN) caring for an HIV-positive patient admitted with tuberculosis can delegate which action to unlicensed assistive personnel (UAP)? (Exam 6) a. Teach the patient about how to use tissues to dispose of respiratory secretions. b. Stock the patients room with all the necessary personal protective equipment. c. Interview the patient to obtain the names of family members and close contacts. d. Tell the patients family members the reason for the use of airborne precautions.

b. Stock the patients room with all the necessary personal protective equipment.

A 73-year-old patient with Parkinson's disease has a nursing diagnosis of impaired physical mobility related to bradykinesia. Which action will the nurse include in the plan of care? (Exam 3) a. Instruct the patient in activities that can be done while lying or sitting. b. Suggest that the patient rock from side to side to initiate leg movement. c. Have the patient take small steps in a straight line directly in front of the feet. d. Teach the patient to keep the feet in contact with the floor and slide them forward.

b. Suggest that the patient rock from side to side to initiate leg movement.

After evacuation of an epidural hematoma, a patient's intracranial pressure (ICP) is being monitored with an intraventricular catheter. Which information obtained by the nurse requires urgent communication with the health care provider? (Exam 3) a. Pulse of 102 beats/min b. Temperature of 101.6°F c. Intracranial pressure of 15 mm Hg d. Mean arterial pressure of 90 mmHg

b. Temperature of 101.6°F

Following a thyroidectomy, a patient reports "a tingling feeling around my mouth." Which assessment should the nurse complete first? (Exam 1) a. Observe for blood on the neck dressing. b. Test for presence of Chvostek's sign. c. Verify the serum potassium level. d. Confirm a prescription for thyroid replacement.

b. Test for presence of Chvostek's sign. Response Feedback: The patient's symptoms indicate possible hypocalcemia, which can occur secondary to parathyroid injury or removal during thyroidectomy. There is no indication of an urgent need to check the potassium level, the thyroid replacement, or for bleeding

A patient who had a total hip replacement had an intraoperative hemorrhage 14 hours ago. Which laboratory result would the nurse expect to find? (Exam 6) a. Hematocrit of 46% b. Hemoglobin of 13.8 g/dL c. Elevated reticulocyte count d. Decreased white blood cell (WBC) count

c. Elevated reticulocyte count

The nurse is caring for a patient with idiopathic pulmonary arterial hypertension (IPAH). Which assessment information requires the most immediate action by the nurse? (Exam 1) a. The O2 saturation is 90%. b. The epoprostenol (Flolan) infusion is disconnected. c. The blood pressure is 98/56 mm Hg. d. The international normalized ratio (INR) is prolonged.

b. The epoprostenol (Flolan) infusion is disconnected. Response Feedback: The half-life of this drug is 6 minutes, so the nurse will need to restart the infusion as soon as possible to prevent rapid clinical deterioration. The other data also indicate a need for ongoing monitoring or intervention, but the priority action is to reconnect the infusion.

The health care provider is considering the use of sumatriptan (Imitrex) for a 54-year-old male patient with migraine headaches. Which information obtained by the nurse is most important to report to the health care provider? (Exam 3) a. The patient drinks 1 to 2 cups of coffee daily. b. The patient had a recent acute myocardial infarction. c. The patient has had migraine headaches for 30 years. d. The patient has taken topiramate (Topamax) for 2 months.

b. The patient had a recent acute myocardial infarction.

The nurse is assessing a patient with osteoarthritis who uses naproxen. Which manifestations shows a need for a change in medication? (Exam 4) a. The patient has gained 3 pounds. b. The patient has dark-colored stools. c. The patient's pain affects multiple joints. d. The patient uses capsaicin cream (Zostrix).

b. The patient has dark-colored stools.

The nurse interviews a patient with a new diagnosis of chronic obstructive pulmonary disease (COPD). Which information is specific in confirming a diagnosis of chronic bronchitis? a. The patient has a 30 pack-year cigarette smoking history. b. The patient reports a productive cough for 3 months of every winter. c. The patient has respiratory problems that began during the past 12 months. d. The patient relates a family history of bronchitis.

b. The patient reports a productive cough for 3 months every winter Response Feedback: A diagnosis of chronic bronchitis is based on a history of having a productive cough for 3 months for at least 2 consecutive years. There is no family tendency for chronic bronchitis. Although smoking is the major risk factor for chronic bronchitis, a smoking history does not confirm the diagnosis.

When caring for a patient after cystoscopy, what should the nurse include in the plan of care? (Exam 1) a. The patient restricts activity to bed rest for 4 to 6 hours. b. The patient understands to expect blood-tinged urine. c. The patient remains NPO for 8 hours to prevent vomiting. d. The patient learns to request narcotics for pain.

b. The patient understands to expect blood-tinged urine. Response Feedback: Pink-tinged urine and urinary frequency are expected after cystoscopy. Burning on urination is common, but pain that requires opioids for relief is not expected. A good fluid intake is encouraged after this procedure. Bed rest is not required after cystoscopy.

The nurse develops a teaching plan to help increase activity tolerance at home for an older adult with severe chronic obstructive pulmonary disease (COPD). Which instructions would be appropriate for the nurse to include in the plan of care? (Exam 1) a. Limit exercise to activities of daily living (ADLs). b. Walk 15 to 20 minutes a day at least 3 times/wk. c. Walk until pulse rate exceeds 130 beats/min. d. Stop exercising when you feel short of breath.

b. Walk 15 to 20 minutes a day at least 3times/wk Encourage the patient to walk 15 to 20 minutes a day at least three times a week with gradual increases. Shortness of breath is normal with exercise and not an indication that the patient should stop. Limiting exercise to ADLs will not improve the patient's exercise tolerance. A 70-yr-old patient should have a pulse rate of 120 beats/min or less with exercise (80% of the maximal heart rate of 150 beats/min).

The son of a dying patient tells the nurse, Mother doesnt really respond any more when I visit. I dont think she knows that I am here. Which response by the nurse is appropriate? (Exam 6) a. You may need to cut back your visits for now to avoid overtiring your mother. b. Withdrawal may sometimes be a normal response in the process of dying c. It will be important for you to stimulate your mother as she gets closer to dying. d. Many patients dont really know what is going on around them at the end of life.

b. Withdrawal may sometimes be a normal response in the process of dying

What findings should the nurse expect in the assessment of a young adult with infective endocarditis (Exam 2) a. substernal chest pressure. b. a new regurgitant murmur. c. a pruritic rash on the chest. d. involuntary muscle movement

b. a new regurgitant murmur

It is important for the nurse providing care for a patient with sickle cell crisis to (Exam 6) a. limit the patients intake of oral and IV fluids. b. evaluate the effectiveness of opioid analgesics. c. encourage the patient to ambulate as much as tolerated. d. teach the patient about high-protein, high-calorie foods.

b. evaluate the effectiveness of opioid analgesics.

The nurse supervises a student nurse who is assigned to take care of a patient with active tuberculosis (TB). Which action, if performed by the student nurse, would require an intervention by the nurse? (Exam 1) a. A snack is brought to the patient from the unit refrigerator. b. A surgical face mask is applied before visiting the patient. c. The patient is offered a tissue from the box at the bedside. d. Hand washing is performed before entering the patient's room.

b. a surgical mask is applied before visiting the patient. Response Feedback: A high-efficiency particulate-absorbing (HEPA) mask, rather than a standard surgical mask, should be used when entering the patient's room because the HEPA mask can filter out 100% of small airborne particles. Hand washing before entering the patient's room is appropriate. Because anorexia and weight loss are frequent problems in patients with TB, bringing food to the patient is appropriate. The student nurse should perform hand washing after handling a tissue that the patient has used, but no precautions are necessary when giving the patient an unused tissue.

A construction worker arrives at an urgent care center with a deep puncture wound from a rusty nail. The patient reports having had a tetanus booster 6 years ago. The nurse will anticipate (Exam 3) a. IV infusion of tetanus immune globulin (TIG). b. administration of the tetanus-diphtheria (Td) booster. c. intradermal injection of an immune globulin test dose. d. initiation of the tetanus-diphtheria immunization series

b. administration of the tetanus-diphtheria (Td) booster.

During the teaching session for a patient who has a new diagnosis of acute leukemia the patient is restless and is looking away, never making eye contact. After teaching about the complications associated with chemotherapy, the patient asks the nurse to repeat all of the information. Based on this assessment, which nursing diagnosis is most appropriate for the patient? (Exam 6) a. denial b. anxiety c. acute confusion d. ineffective adherence to treatment

b. anxiety

The nurse is caring for a client with a diagnosis of gout. Which lab value would the nurse expect to note in the client? (Exam 4)

uric acid level of 9.0 mg/dL

To assess the patient with pericarditis for evidence of a pericardial friction rub, the nurse should a. listen for a rumbling, low-pitched, systolic murmur over the left anterior chest. (exam 2) b. auscultate by placing the diaphragm of the stethoscope on the lower left sternal border. c. ask the patient to cough during auscultation to distinguish the sound from a pleural friction rub. d. feel the precordial area with the palm of the hand to detect vibrations with cardiac contraction.

b. auscultate by placing the diaphragm of the stethoscope on the lower left sternal border.

What risks will the nurse plan to teach a 27-year-old woman who smokes two packs of cigarettes daily? (Exam 1) a. interstitial cystitis b. bladder cancer c. bladder infection d. kidney stones

b. bladder cancer Cigarette smoking is a risk factor for bladder cancer. The patient's risk for developing interstitial cystitis, urinary tract infection, or kidney stones will not be reduced by quitting smoking.

A 28-year-old man with von Willebrand disease is admitted to the hospital for minor knee surgery. The nurse will review the coagulation survey to check the a. platelet count. b. bleeding time. c. thrombin time. d. prothrombin time. (Exam 6)

b. bleeding time.

A routine complete blood count indicates that an active 80-year-old man may have myelodysplastic syndrome. The nurse will plan to teach the patient about (Exam 6) a. blood transfusion b. bone marrow biopsy. c. filgrastim (Neupogen) administration. d. erythropoietin (Epogen) administration.

b. bone marrow biopsy.

When obtaining a health history and physical assessment for a 36-year-old female patient with possible multiple sclerosis (MS), the nurse should (Exam 3) a. assess for the presence of chest pain. b. inquire about urinary tract problems. c. inspect the skin for rashes or discoloration. d. ask the patient about any increase in libido.

b. inquire about urinary tract problems.

The priority nursing assessment for a patient being admitted with a brainstem infarction is (Exam 3) a. pupil reaction. b. respiratory rate. c. reflex reaction time. d. level of consciousness.

b. respiratory rate.

To prevent recurrence of uric acid kidney stones, the nurse teaches the patient to avoid eating: (Exam 1) a. legumes and dried fruit. b. sardines and liver. c. milk and cheese. d. spinach and chocolate.

b. sardines and liver Response Feedback: Organ meats and fish such as sardines increase purine levels and uric acid. Spinach, chocolate, and tomatoes should be avoided in patients who have oxalate stones. Milk, dairy products, legumes, and dried fruits may increase the incidence of calcium-containing stones.

A client is complaining of low back pain that radiates down the left posterior thigh. The nurse would ask the client if the pain is worsened or aggravated by which factor? (Exam 4)

bending of lifting

A patient is admitted to the emergency department with severe fatigue and confusion. Laboratory studies are done. Which laboratory value will require the most immediate action by the nurse? a. Arterial blood pH is 7.32. b. Serum calcium is 18 mg/dL. c. Arterial oxygen saturation is 91%. d. Serum potassium is 5.1 mEq/L.

b. serum calcium is 18 mg/dL Response Feedback: The serum calcium is well above the normal level and puts the patient at risk for cardiac dysrhythmias. The nurse should start cardiac monitoring and notify the health care provider. The potassium, oxygen saturation, and pH are also abnormal, and the nurse should notify the health care provider about these values as well, but they are not immediately life threatening.

The nurse assessing a patient with newly diagnosed trigeminal neuralgia will ask the patient about (Exam 3) a. visual problems caused by ptosis. b. triggers leading to facial discomfort. c. poor appetite caused by loss of taste. d. weakness on the affected side of the face.

b. triggers leading to facial discomfort.

The nurse is caring for a patient diagnosed with stage I colon cancer. When assessing the need for psychologic support, which question by the nurse will provide the most information? (Exam 6) a. "How long ago were you diagnosed with this cancer?" b. "Do you have any concerns about body image changes?" c. "Can you tell me what has been helpful to you in the past when coping with stressful events?" d. "Are you familiar with the stages of emotional adjustment to a diagnosis like cancer of the colon?"

c. "Can you tell me what has been helpful to you in the past when coping with stressful events?"

A high school teacher with ulnar drift caused by rheumatoid arthritis is scheduled for arthroplasty of several joints in the left hand. Which patient statement to the nurse indicates a realistic expectation for the surgery? (Exam 4) a. "This procedure will correct the deformities in my fingers." b. "I will not have to do as many hand exercises to grasp things." c. "I will be able to use my fingers with more flexibility to grasp things." d. "My fingers will appear more normal in size and shape after this surgery."

c. "I will be able to use my fingers with more flexibility to grasp things."

A patient with heart failure has a new order for captopril 12.5 mg PO. After giving the first dose and teaching the patient about the drug, which statement by the patient indicates that teaching has been effective? (Exam 2) a. "I will be sure to take the medication with food." b. "I will need to eat more potassium-rich foods in my diet." c. "I will call for help when I need to get up to use the bathroom." d. "I will expect to feel more short of breath for the next few days."

c. "I will call for help when I need to get up to use the bathroom."

The nurse teaches a patient with Osteoarthritis of the hip about how to manage the OA. Which patient statement indicates to the nurse a need for additional teaching? (Exam 4) a. "A shower in the morning will help relieve stiffness." b. "I can exercise every day to help maintain my mobility." c. "I will take 1 gram of acetaminophen (Tylenol) every 4 hours."

c. "I will take 1 gram of acetaminophen (Tylenol) every 4 hours."

A student asks the nurse why a peripherally inserted central catheter is needed for a patient receiving parenteral nutrition with 25% dextrose. Which response by the nurse is accurate?(Exam 1) a. "There is a decreased risk for infection when 25% dextrose is infused through a central line." b. "The required blood glucose monitoring is based on samples obtained from a central line." c. "The hypertonic solution will be more rapidly diluted when given through a central line." d. "The prescribed infusion can be given more rapidly when the patient has a central line."

c. "The hypertonic solution will be more rapidly diluted when given through a central line." Response Feedback: The 25% dextrose solution is hypertonic. Shrinkage of red blood cells can occur when solutions with dextrose concentrations greater than 10% are administered IV. Blood glucose testing is not more accurate when samples are obtained from a central line. The infection risk is higher with a central catheter than with peripheral IV lines. Hypertonic or concentrated IV solutions are not given rapidly.

A patient with ST-segment elevation in three contiguous electrocardiographic leads is admitted to the emergency department and diagnosed as having an ST-segment-elevation myocardial infarction (STEMI). Which question should the nurse ask to determine whether the patient is a candidate for thrombolytic therapy? (Exam 2) a. "Do you have any allergies?" b. "Do you take aspirin daily?" c. "What time did your pain begin?" d. "Can you rate the pain on a 0 to 10 scale?

c. "What time did your pain begin?"

The nurse receives change-of-shift report on the oncology unit. Which patient should the nurse assess first? (Exam 6) a. 35-year-old patient who has wet desquamation associated with abdominal radiation b. 42-year-old patient who is sobbing after receiving a new diagnosis of ovarian cancer c. 24-year-old patient who received neck radiation and has blood oozing from the neck d. 56-year-old patient who developed a new pericardial friction rub after chest radiation

c. 24-year-old patient who received neck radiation and has blood oozing from the neck

Immediately after the nurse administers an intracutaneous injection of an allergen on the forearm, a patient complains of itching at the site and of weakness and dizziness. What action should the nurse take first? (Exam 6) a. Remind the patient to remain calm. b. Administer subcutaneous epinephrine. c. Apply a tourniquet above the site. d. Rub a local antiinflammatory cream on the site.

c. Apply a tourniquet above the site.

The nurse assesses that a patient receiving epidural morphine has not voided for over 10 hours. What action should the nurse take initially? (Exam 6) a. Monitor for withdrawal symptoms. b. Place an indwelling urinary catheter. c. Ask if the patient feels the need to void. d. Document this allergic reaction in the patients chart.

c. Ask if the patient feels the need to void.

A patient admitted with dermal ulcers who has a history of a T3 spinal cord injury tells the nurse, "I have a pounding headache and I feel sick to my stomach." Which action should the nurse take first? (Exam 3) a. Check for a fecal impaction. b. Give the prescribed analgesic. c. Assess the blood pressure (BP). d. Notify the health care provider.

c. Assess the blood pressure

A new mother expresses concern about her baby developing allergies and asks what the health care provider meant by passive immunity. Which example should the nurse use to explain this type of immunity? (Exam 6) a. Early immunization b. Bone marrow donation c. Breastfeeding her infant d. Exposure to communicable diseases

c. Breastfeeding her infant

What should the nurse include when teaching older adults at a community recreation center about ways to prevent fractures? (Exam 4) a. Tack down scatter rugs on the floor in the home b. Expect most falls to happen outside the home in the yard c. Buy shoes that provide good support and are comfortable to wear

c. Buy shoes that provide good support and are comfortable to wear

The nurse should anticipate the need to teach a patient who has osteoarthriritis about which medication? (Exam 4) a. Prednisone b. Adalimumab (humira) c. Capsaicin cream (Zostrix) d. Sulfasalazine

c. Capsaicin cream (Zostrix)

A 70-year-old female patient with left-sided hemiparesis arrives by ambulance to the emergency department. Which action should the nurse take first? (Exam 3) a. Monitor the blood pressure. b. Send the patient for a computed tomography (CT) scan. c. Check the respiratory rate and effort. d. Assess the Glasgow Coma Scale score.

c. Check the respiratory rate and effort.

External-beam radiation is planned for a patient with cervical cancer. What instructions should the nurse give to the patient to prevent complications from the effects of the radiation? (Exam 6) a. Test all stools for the presence of blood. b. Maintain a high-residue, high-fiber diet. c. Clean the perianal area carefully after every bowel movement. d. Inspect the mouth and throat daily for the appearance of thrush.

c. Clean the perianal area carefully after every bowel movement.

Diltiazem (Cardizem) is prescribed for a patient with newly diagnosed Prinzmetal's (variant) angina. Which action of diltiazem is accurate for the nurse to include in the teaching plan? (Exam 2) a. Reduces heart palpitations. b. Prevents coronary artery plaque. c. Decreases coronary artery spasms. d. Increases contractile force of the heart.

c. Decreases coronary artery spasms.

Which finding in a patient with a spinal cord tumor is most important for the nurse to report to the health care provider? (Exam 3) a. Back pain that increases with coughing b. Depression about the diagnosis of a tumor c. Decreasing sensation and ability to move the legs d. Anxiety about scheduled surgery to remove the tumor

c. Decreasing sensation and ability to move the legs

A patient treated for human immunodeficiency virus (HIV) infection for 6 years has developed fat redistribution to the trunk, with wasting of the arms, legs, and face. What instructions will the nurse give to the patient? (Exam 6) a. Review foods that are higher in protein. b. Teach about the benefits of daily exercise. c. Discuss a change in antiretroviral therapy. d. Talk about treatment with antifungal agents.

c. Discuss a change in antiretroviral therapy.

Which action can the registered nurse (RN) who is caring for a critically ill patient with multiple IV lines and medications delegate to a licensed practical/vocational nurse (LPN/VN)? (Exam 1) a. Verify blood products prior to administration. b. Remove the patient's central venous catheter. c. Flush a saline lock with normal saline. d. Titrate vasoactive IV medications.

c. Flush a saline lock with normal saline Response Feedback: A LPN/VN has the education, experience, and scope of practice to flush a saline lock with normal saline. Administration of blood products, adjustment of vasoactive infusion rates, and removal of central catheters in critically ill patients require RN level education and scope of practice.

A patient had a cystectomy with an ileal conduit yesterday. Which new assessment data is most important for the nurse to communicate to the health care provider? (Exam 1) a. Cloudy appearing urine b. Hypoactive bowel sounds c. Heart rate 102 beats/min d. Continuous stoma drainage

c. Heart rate 102 beats/min Response Feedback: Tachycardia may indicate infection, hemorrhage, or hypovolemia, which are all serious complications of this surgery. The urine from an ileal conduit normally contains mucus and is cloudy. Hypoactive bowel sounds are expected after bowel surgery. Continuous drainage of urine from the stoma is normal.

A patient who is unconscious has ineffective cerebral tissue perfusion and cerebral tissue swelling. Which nursing intervention will be included in the plan of care? (Exam 3) a. Encourage coughing and deep breathing. b. Position the patient with knees and hips flexed. c. Keep the head of the bed elevated to 30 degrees. d. Cluster nursing interventions to provide rest periods.

c. Keep the head of the bed elevated to 30 degrees.

A patient who is receiving dobutamine for the treatment of acute decompensated heart failure (ADHF) has the following nursing interventions included in the plan of care. Which action will be most appropriate for the registered nurse (RN) to delegate to an experienced licensed practical/vocational nurse (LPN/LVN)? (Exam 2) a. Teach the patient the reasons for remaining on bed rest. b. Change the peripheral IV site according to agency policy. c. Monitor the patient's blood pressure and heart rate every hour. d. Titrate the rate to keep the systolic blood pressure >90 mm Hg

c. Monitor the patient's blood pressure and heart rate every hour.

A patient recovering from heart surgery develops pericarditis and complains of level 6 (0 to 10 scale) chest pain with deep breathing. Which ordered PRN medication will be the most appropriate for the nurse to give? (Exam 2) a. Fentanyl 1 mg IV b. IV morphine sulfate 4 mg c. Oral ibuprofen (Motrin) 600 mg d. Oral acetaminophen (Tylenol) 650 mg

c. Oral ibuprofen (Motrin) 600 mg

Which patient should the nurse assess first? (Exam 6) a. Patient with urticaria after receiving an IV antibiotic b. Patient who has graft-versus-host disease and severe diarrhea c. Patient who is sneezing after having subcutaneous immunotherapy d. Patient with multiple chemical sensitivities who has muscle stiffness

c. Patient who is sneezing after having subcutaneous immunotherapy

Which hospitalized patient will the nurse assign to the room closest to the nurses' station? (Exam 3) a. Patient with Alzheimer's disease who has long-term memory deficit b. Patient with vascular dementia who takes medications for depression c. Patient with new-onset confusion, restlessness, and irritability after surgery d. Patient with dementia who has an abnormal Mini-Mental State Examination

c. Patient with new-onset confusion, restlessness, and irritability after surgery

A nurse provides care to a client who has a fractured femur after falling from a ladder. Which of the following actions may reduce the incidence of fat emboli? (Exam 4)

immobilize the extremity

When caring for a patient with a new right-sided homonymous hemianopsia resulting from a stroke, which intervention should the nurse include in the plan of care? (Exam 3) a. Apply an eye patch to the right eye. b. Approach the patient from the right side. c. Place objects needed on the patient's left side. d. Teach the patient that the left visual deficit will resolve.

c. Place objects needed on the patient's left side.

Which electrocardiographic (ECG) change by a patient with chest pain is most important for the nurse to report rapidly to the health care provider? (Exam 2) a. Inverted P wave b. Sinus tachycardia c. ST-segment elevation d. First-degree atrioventricular block

c. ST-segment elevation

The nurse is caring for a patient with left-sided lung cancer. Which finding would be most important for the nurse to report to the health care provider? (Exam 6) a. Hematocrit 32% b. Pain with deep inspiration c. Serum sodium 126 mEq/L d. Decreased breath sounds on left side

c. Serum sodium 126 mEq/L

A patient with renal failure is on a low phosphate diet. Which food should the nurse instruct unlicensed assistive personnel (UAP) to remove from the patient's food tray? (Exam 1) a. Fried chicken breast b. Grape juice c. Skim milk d. Mixed green salad

c. Skim milk Response Feedback: Foods high in phosphate include milk and other dairy products, so these are restricted on low-phosphate diets. Green, leafy vegetables; high-fat foods; and fruits and juices are not high in phosphate and are not restricted.

A patient from a long-term care facility is admitted to the hospital with a sacral pressure ulcer. The base of the wound involves subcutaneous tissue. How should the nurse classify this pressure ulcer? (Exam 6) a. Stage I b. Stage 2 c. Stage 3 d. Stage 4

c. Stage 3

A patient admitted with possible stroke has been aphasic for 3 hours and has a current blood pressure of 174/94 mm Hg. Which order by the health care provider should the nurse question? (Exam 3) a. Keep head of bed elevated at least 30 degrees. b. Infuse normal saline intravenously at 75 mL/hr. c. Start a labetalol drip to keep BP less than 140/90 mm Hg. d. Begin tissue plasminogen activator (tPA) intravenously per protocol.

c. Start a labetalol drip to keep BP less than 140/90 mm Hg.

A 55-yr-old woman admitted for shoulder surgery asks the nurse for a perineal pad, stating that laughing or coughing causes leakage of urine. Which intervention is appropriate to include in the care plan? (Exam 1) a. Place a commode at the patient's bedside. b. Demonstrate how to perform the Credé maneuver. c. Teach the patient how to perform Kegel exercises. d. Assist the patient to the bathroom q3hr.

c. Teach the patient how to perform Kegel exercises. Response Feedback: Kegel exercises to strengthen the pelvic floor muscles will help reduce stress incontinence. The Credé maneuver is used to help empty the bladder for patients with overflow incontinence. Placing the commode close to the bedside and assisting the patient to the bathroom are helpful for functional incontinence.

Which information about a patient with MS indicates that the nurse should consult with the health care provider before giving the prescribed dose of dalfampridine (Ampyra)? (Exam 3) a. The patient has relapsing-remitting MS. b. The patient enjoys walking for relaxation. c. The patient has an increased creatinine level. d. The patient complains of pain with neck flexion.

c. The patient has an increased creatinine level

Which information obtained during the nurse's assessment may indicate a patient's increased risk for musculoskeletal problems? (Exam 4) a. The patient takes a multivitamin daily b. The patient dislikes fruits and vegetables c. The patient is 5 ft, 2 in tall and weighs 180 pounds d. The patient prefers whole milk to nonfat milk

c. The patient is 5 ft, 2 in tall and weighs 180 pounds

The nurse is caring for a patient who has a head injury and fractured right arm after being assaulted. Which assessment information requires the most rapid action by the nurse? (Exam 3) a. The apical pulse is slightly irregular. b. The patient complains of a headache. c. The patient is more difficult to arouse. d. The blood pressure (BP) increases to 140/62 mm Hg.

c. The patient is more difficult to arouse.

The nurse observes a student who is listening to a patient's lungs. Which action by the student indicates a need to review respiratory assessment skills? (Exam 1) a. The student instructs the patient to breathe slowly and deeply through the mouth b. The student compares breath sounds from side to side at each level. c. The student listens during the inspiratory phase, then moves the stethoscope d. The student starts at the apices of the lungs, moving down toward the lung bases

c. The student listens during the inspiratory phase, then moves the stethoscope Listening only during inspiration indicates the student needs a review of respiratory assessment skills. At each placement of the stethoscope, listen to at least one cycle of inspiration and expiration. During chest auscultation, instruct the patient to breathe slowly and a little deeper than normal through the mouth. Auscultation should proceed from the lung apices to the bases, comparing opposite areas of the chest, unless the patient is in respiratory distress or will tire easily.

While the nurse is transporting a patient on a stretcher to the radiology department, the patient begins having a tonic-clonic seizure. Which action should the nurse take? (Exam 3) a. Insert an oral airway during the seizure to maintain a patent airway. b. Restrain the patient's arms and legs to prevent injury during the seizure. c. Time and observe and record the details of the seizure and postictal state. d. Avoid touching the patient to prevent further nervous system stimulation.

c. Time and observe and record the details of the seizure and postictal state.

When a patient arrives in the ED with a facial fracture, which action should the nurse take first? (Exam 4) a. Assess for nasal bleeding and pain b. Apply ice to the face to reduce swelling c. Use a cervical collar to stabilize the spine

c. Use a cervical collar to stabilize the spine

Which discharge instruction should the emergency department nurse include for a patient with a sprained ankle? (Exam 4) a. Keep the ankle loosely wrapped with gauze b. Apply a heating pad to reduce muscle spasms c. Use pillows to elevate the ankle above the heart d. Gently move the ankle through the range of motion

c. Use pillows to elevate the ankle above the heart

The nurse is watching the cardiac monitor and notices that the rhythm suddenly changes. There are no P waves, the QRS complexes are wide, and the ventricular rate is regular, but over 100 bpm. The nurse determines that the client is experiencing which of the following dysrhythmias? a. Sinus tachycardia b. Ventricular fibrillation c. Ventricular tachycardia d. Premature ventricular contractions (PVCs)

c. Ventricular tachycardia

An older adult is receiving standard multidrug therapy for tuberculosis (TB). Which finding should the nurse report to the health care provider? (Exam 1) a. Thickening of the fingernails b. Difficulty hearing high-pitched voices c. Yellow-tinged sclera d. Orange-colored sputum

c. Yellow-tinged sclera Response Feedback: Noninfectious hepatitis is a toxic effect of isoniazid, rifampin, and pyrazinamide, and patients who develop hepatotoxicity will need to use other medications. Changes in hearing and nail thickening are not expected with the four medications used for initial TB drug therapy. Presbycusis is an expected finding in the older adult patient. Orange discoloration of body fluids is an expected side effect of rifampin and not an indication to call the health care provider.

The nurse performing a focused assessment of left posterior temporal lobe functions will assess the patient for (Exam 3) a. sensation on the left side of the body. b. reasoning and problem-solving ability. c. ability to understand written and oral language. d. voluntary movements on the right side of the body.

c. ability to understand written and oral language.

After ureterolithotomy, a patient has a left ureteral catheter and a urethral catheter in place. Which action will the nurse include in the plan of care? (Exam 1) a. Clamp the ureteral catheter off when output from the urethral catheter stops. b. Apply continuous steady tension to the ureteral catheter. c. Call the health care provider if the ureteral catheter output drops suddenly. d. Provide teaching about home care for both catheters.

c. call the healthcare provider if the ureteral catheter output drops suddenly Response Feedback: The health care provider should be notified if the ureteral catheter output decreases because obstruction of this catheter may result in an increase in pressure in the renal pelvis. Tension on the ureteral catheter should be avoided to prevent catheter displacement. To avoid pressure in the renal pelvis, the catheter is not clamped. Because the patient is not usually discharged with a ureteral catheter in place, patient teaching about both catheters is not needed.

When a brain-injured patient responds to nail bed pressure with internal rotation, adduction, and flexion of the arms, the nurse reports the response as (Exam 3) a. flexion withdrawal. b. localization of pain. c. decorticate posturing. d. decerebrate posturing

c. decorticate posturing

A 38-year-old patient has returned home following rehabilitation for a spinal cord injury. The home care nurse notes that the spouse is performing many of the activities that the patient had been managing unassisted during rehabilitation. The most appropriate action by the nurse at this time is to (Exam 3) a. remind the patient about the importance of independence in daily activities. b. tell the spouse to stop because the patient is able to perform activities independently. c. develop a plan to increase the patient's independence in consultation with the patient and the spouse. d. recognize that it is important for the spouse to be involved in the patient's care and encourage that participation.

c. develop a plan to increase the patient's independence in consultation with the patient and the spouse.

A nurse is assessing a newly admitted patient with chronic heart failure who forgot to take prescribed medications. The patient seems confused and short of breath with peripheral edema. Which assessment should the nurse complete first? (Exam 1) a. skin turgor b. capillary refill c. mental status d. heart sounds

c. mental status Increases in extracellular fluid (ECF) can lead to swelling of cells in the central nervous system, initially causing confusion, which may progress to coma or seizures. Although skin turgor, capillary refill, and heart sounds may also be affected by increases in ECF, these are signs that do not have as immediate impact on patient outcomes as cerebral edema

A patient is hospitalized with new onset of Guillain-Barré syndrome. The most essential assessment for the nurse to complete is (Exam 3) a. determining level of consciousness. b. checking strength of the extremities. c. observing respiratory rate and effort. d. monitoring the cardiac rate and rhythm.

c. observing respiratory rate and effort.

A patient who has chronic heart failure tells the nurse, "I was fine when I went to bed, but I woke up in the middle of the night feeling like I was suffocating!" The nurse will document this assessment finding as: (Exam 2) a. orthopnea. b. pulsus alternans. c. paroxysmal nocturnal dyspnea. d. acute bilateral pleural effusion.

c. paroxysmal nocturnal dyspnea.

A patient who had a C7 spinal cord injury a week ago has a weak cough effort and audible rhonchi. The initial intervention by the nurse should be to (Exam 3) a. administer humidified oxygen by mask. b. suction the patient's mouth and nasopharynx. c. push upward on the epigastric area as the patient coughs. d. encourage incentive spirometry every 2 hours during the day.

c. push upward on the epigastric area as the patient coughs.

To assess the functions of the trigeminal and facial nerves (CNs V and VII), the nurse should (Exam 3) a. check for unilateral eyelid droop. b. shine a light into the patient's pupil. c. touch a cotton wisp strand to the cornea. d. have the patient read a magazine or book.

c. touch a cotton wisp strand to the cornea.

During the change of shift report a nurse is told that a patient has an occluded left posterior cerebral artery. The nurse will anticipate that the patient may have (Exam 3) a. dysphasia. b. confusion. c. visual deficits. d. poor judgment

c. visual deficits.

Which information will the nurse include when teaching a patient who is scheduled for a radiofrequency catheter ablation for treatment of atrial flutter? (Exam 2) a.The procedure will prevent or minimize the risk for sudden cardiac death. b.The procedure will use cold therapy to stop the formation of the flutter waves. c.The procedure will use electrical energy to destroy areas of the conduction system. d.The procedure will stimulate the growth of new conduction pathways between the atria

c.The procedure will use electrical energy to destroy areas of the conduction system

A patient is scheduled for a cardiac catheterization with coronary angiography. Before the test, the nurse informs the patient that (Exam 2) a. it will be important not to move at all during the procedure. b. monitored anesthesia care will be provided during the procedure. c.a flushed feeling may be noticed when the contrast dye is injected. d. arterial pressure monitoring will be required for 24 hours after the test.

c.a flushed feeling may be noticed when the contrast dye is injected.

The nurse is providing care for a client diagnosed with invasive pancreatic cancer. The client has a permanent biliary drainage tube (T-tube) inserted to provide palliative care. Which action should the nurse take postoperatively? (Exam 5)

cleanse the area around the insertion site to prevent skin breakdown

The nurse is caring for a client being treated for a fat embolus after multiple fractures. Which data would the RN determine is the most favorable indication of resolution of the fat embolus? (Exam 4)

clear mentation

Which patient statement indicates that the nurse's teaching about sublingual nitro has been effective?(Exam 2) a. "I can expect nausea as a side effect of nitroglycerin." b. "I should only take nitroglycerin when I have chest pain." c. "Nitroglycerin helps prevent a clot from forming and blocking blood flow to my heart." d. "I will call an ambulance if I have pain after taking 3 nitroglycerin 5 minutes apart

d. "I will call an ambulance if I have pain after taking 3 nitroglycerin 5 minutes apart

The nurse provides discharge teaching for a patient who has two fractured ribs from an automobile accident. Which patient statement indicates that teaching has been effective? (Exam 1) a. "I am going to buy a rib binder to wear during the day." b. "I should plan on taking the pain pills only at bedtime so I can sleep." c. "I can take shallow breaths to prevent my chest from hurting." d. "I will use the incentive spirometer every hour or two during the day."

d. "I will use the incentive spirometer every hour or two during the day." Response Feedback: Prevention of the complications of atelectasis and pneumonia is a priority after rib fracture. This can be ensured by deep breathing and coughing. Use of a rib binder, shallow breathing, and taking pain medications only at night are likely to result in atelectasis.

The nurse is caring for a patient with mitral regurgitation. Referring to the figure below, where should the nurse listen to best hear any murmur that the patient has? (Exam 2) a. 1 b. 2 c. 3 d. 4 - apex of the heart

d. 4 - apex of the heart

After the emergency department nurse has received a status report on the following patients who have been admitted with head injuries, which patient should the nurse assess first? (Exam 3) a. A 20-yr-old patient whose cranial x-ray shows a linear skull fracture b. A 50-yr-old patient who has an initial Glasgow Coma Scale score of 13 c. A 30-yr-old patient who lost consciousness for a few seconds after a fall d. A 50-yr-old patient whose right pupil is 10 mm and unresponsive to light

d. A 50-yr-old patient whose right pupil is 10 mm and unresponsive to light

After receiving change-of-shift report about the following four patients on the cardiac care unit, which patient should the nurse assess first? (Exam 2) a. A 39-yr-old patient with pericarditis who is complaining of sharp, stabbing chest pain b. A 56-yr-old patient with variant angina who is scheduled to receive nifedipine (Procardia) c. A 65-yr-old patient who had a myocardial infarction (MI) 4 days ago and is anxious about today's planned discharge d. A 59-yr-old patient with unstable angina who has just returned after a percutaneous coronary intervention (PCI)

d. A 59-yr-old patient with unstable angina who has just returned after a percutaneous coronary intervention (PCI)

Which laboratory test will the nurse use to determine whether filgrastim (Neupogen) is effective for a patient with acute lymphocytic leukemia who is receiving chemotherapy? (Exam 6) a. Platelet count b. Reticulocyte count c. Total lymphocyte count d. Absolute neutrophil count

d. Absolute neutrophil count

Which prescribed intervention will the nurse implement first for a patient in the emergency department who is experiencing continuous tonic-clonic seizures? (Exam 3) a. Give phenytoin (Dilantin) 100 mg IV. b. Monitor level of consciousness (LOC). c. Obtain computed tomography (CT) scan. d. Administer lorazepam (Ativan) 4 mg IV.

d. Administer lorazepam (Ativan) 4 mg IV

A patient with rheumatoid arthritis has been taking corticosteroids for 11 months. Which nursing action is most likely to detect early signs of infection in this patient? (Exam 6) a. Monitor white blood cell count. b. Check the skin for areas of redness. c. Check the temperature every 2 hours. d. Ask about fatigue or feelings of malaise.

d. Ask about fatigue or feelings of malaise.

Which topic will the nurse plan to include in discharge teaching for a patient with heart failure with reduced ejection fraction (HFrEF)? (Exam 2) a. Need to begin an aerobic exercise program several times weekly b. Use of salt substitutes to replace table salt when cooking and at the table c. Importance of making an annual appointment with the health care provider d. Benefits and side effects of angiotensin-converting enzyme (ACE) inhibitors

d. Benefits and side effects of angiotensin-converting enzyme (ACE) inhibitors

A patient who has just been admitted with pulmonary edema is scheduled to receive the following medications. Which medication should the nurse question before giving. (Exam 2) a. Furosemide (Lasix) 60 mg b. Captopril (Capoten) 25 mg c. Digoxin (Lanoxin) 0.125 mg d. Carvedilol (Coreg) 3.125 mg

d. Carvedilol (Coreg) 3.125 mg

Which action should the nurse take before administering gentamicin to a patient with acute osteomyelitis? (Exam 4) a. Ask the patient about any nausea b. Obtain the patient's oral temperature c. Change the prescribed wet-to-dry dressings d. Review the patient's serum creatinine results

d. Review the patient's serum creatinine results

A young male patient who is a paraplegic has a stage II sacral pressure ulcer and is being cared for at home by his mother. To prevent further tissue damage, what instructions are most important for the nurse to teach the mother? (Exam 6) a. Change the patient's bedding frequently. b. Use a hydrocolloid dressing over the ulcer. c. Record the size and appearance of the ulcer weekly. d. Change the patient's position at least every 2 hours.

d. Change the patient's position at least every 2 hours.

A patient admitted to the coronary care unit (CCU) with an ST-segment-elevation myocardial infarction (STEMI) is restless and anxious. The blood pressure is 86/40 mm Hg, and heart rate is 132 beats/min. Based on this information, which nursing diagnosis is a priority for the patient? (exam 2) a. Acute pain related to myocardial infarction b. Anxiety related to perceived threat of death c. Stress overload related to acute change in health d. Decreased cardiac output related to cardiogenic shock

d. Decreased cardiac output related to cardiogenic shock

A patient who has been receiving diuretic therapy is admitted to the emergency department with a serum potassium level of 3.0 mEq/L. The nurse should alert the health care provider immediately that the patient is on which medication? (Exam 1) a. Ibuprofen 400 mg every 6 hours b. Lantus insulin 24 U every evening c. Metoprolol (Lopressor) 12.5 mg/day

d. Digoxin (Lanoxin) 0.25 mg/day Response Feedback: Hypokalemia increases the risk for digoxin toxicity, which can cause serious dysrhythmias. The nurse will need to do more assessment about the other medications, but they are not of as much concern with the potassium level.

Which medication taken by a patient with restless legs syndrome should the nurse discuss with the patient? (Exam 3) a. Ibuprofen b. Multivitamin c. Acetaminophen d. Diphenhydramine

d. Diphenhydramine

A patient with pancytopenia has a bone marrow aspiration from the left posterior iliac crest. Which action would be important for the nurse to take after the procedure? (Exam 6) a. Elevate the head of the bed to 45 degrees. b. Apply a sterile 2-inch gauze dressing to the site. c. Use a half-inch sterile gauze to pack the wound. d. Have the patient lie on the left side for 1 hour.

d. Have the patient lie on the left side for 1 hour.

When assessing a newly admitted patient, the nurse notes pallor of the skin and nail beds. The nurse should ensure that which laboratory test has been ordered? (Exam 6) a. Platelet count b. Neutrophil count c. White blood cell count d. Hemoglobin (Hgb) level

d. Hemoglobin (Hgb) level

Following successful treatment of Hodgkin's lymphoma for a 55-yr-old woman, which topic will the nurse include in patient teaching? (Exam 6) a. Potential impact of chemotherapy treatment on fertility b. Application of soothing lotions to treat residual pruritus c. Use of maintenance chemotherapy to maintain remission d. Need for follow-up appointments to screen for malignancy

d. Need for follow-up appointments to screen for malignancy

A patient with left-sided weakness that started 60 minutes earlier is admitted to the emergency department and diagnostic tests are ordered. Which test should be done first? (Exam 3) a. Complete blood count (CBC) b. Chest radiograph (Chest x-ray) c. 12-Lead electrocardiogram (ECG) d. Noncontrast computed tomography (CT)

d. Noncontrast computed tomography (CT)

The nurse is caring for a 70-yr-old patient who uses hydrochlorothiazide and enalapril (Norvasc) but whose self-monitored blood pressure (BP) continues to be elevated. Which patient information may indicate a need for a change?(Exam 2) a. Patient takes a daily multivitamin tablet. b. Patient checks BP daily just after getting up. c. Patient drinks wine three to four times a week. d. Patient uses ibuprofen (Motrin) treat osteoarthritis.

d. Patient uses ibuprofen (Motrin) treat osteoarthritis.

A patient with human immunodeficiency virus (HIV) infection has developed Mycobacterium avium complex infection. Which outcome would be appropriate for the nurse to include in the plan of care? (Exam 6) a. The patient will be free from injury. b. The patient will receive immunizations. c. The patient will have adequate oxygenation. d. The patient will maintain intact perineal skin.

d. The patient will maintain intact perineal skin.

After change-of-shift report on the Alzheimer's disease/dementia unit, which patient will the nurse assess first? (Exam 3) a. Patient who has not had a bowel movement for 5 days b. Patient who has a stage II pressure ulcer on the coccyx c. Patient who is refusing to take the prescribed medications d. Patient who developed a new cough after eating breakfast

d. Patient who developed a new cough after eating breakfast

When a patient with hypertension who has a new prescription for atenolol (Tenormin) returns to the health clinic after 2 weeks for a follow-up visit, the BP is unchanged from the previous visit. Which action should the nurse take first? (Exam 2) a. Provide information about the use of multiple drugs to treat hypertension. b. Teach the patient about the reasons for a possible change in drug therapy. c. Remind the patient that lifestyle changes also are important in BP control. d. Question the patient about whether the medication is actually being taken.

d. Question the patient about whether the medication is actually being taken.

The nurse is caring for a patient who was admitted the previous day with a basilar skull fracture after a motor vehicle crash. Which assessment finding is most important to report to the health care provider? a. Complaint of severe headache b. Large contusion behind left ear c. Bilateral periorbital ecchymosis d. Temperature of 101.4° F (38.6° C) (Exam 3)

d. Temperature of 101.4° F (38.6° C)

A patient in the emergency department with sudden-onset right-sided weakness is diagnosed with an intracerebral hemorrhage. Which information about the patient is most important to communicate to the health care provider? (Exam 3) a. The patient's speech is difficult to understand. b. The patient's blood pressure is 144/90 mm Hg. c. The patient takes a diuretic because of a history of hypertension. d. The patient has atrial fibrillation and takes warfarin (Coumadin).

d. The patient has atrial fibrillation and takes warfarin (Coumadin).

The nurse is obtaining a health history from a 24-year-old patient with hypertrophic cardiomyopathy (HC). Which information obtained by the nurse is most important? (Exam 2) a. The patient has a history of a recent upper respiratory infection. b. The patient has a family history of coronary artery disease (CAD). c. The patient reports using cocaine a "couple of times" as a teenager. d. The patient's 29-year-old brother died from a sudden cardiac arrest

d. The patient's 29-year-old brother died from a sudden cardiac arrest

When assessing a 53-year-old patient with bacterial meningitis, the nurse obtains the following data. Which finding should be reported immediately to the health care provider? (Exam 3) a. The patient exhibits nuchal rigidity. b. The patient has a positive Kernig's sign. c. The patient's temperature is 101° F (38.3° C). d. The patient's blood pressure is 88/42 mm Hg.

d. The patient's blood pressure is 88/42 mm Hg.

The nurse is caring for a mechanically ventilated patient with a cuffed tracheostomy tube. Which action by the nurse would determine if the cuff has been properly inflated? (Exam 1) a. Insert the decannulation plug before removing the nonfenestrated inner cannula. b. Review the health record for the prescribed cuff pressure. c. Suction the patient through a fenestrated inner cannula to clear secretions. d. Use a hand-held manometer to measure cuff pressure.

d. Use a hand-held manometer to measure cuff pressure. Response Feedback: Measurement of cuff pressure using a manometer to ensure that cuff pressure is 20 mm Hg or lower will avoid compression of the tracheal wall and capillaries. Never insert the decannulation plug in a tracheostomy tube until the cuff is deflated and the nonfenestrated inner cannula is removed. Otherwise, the patient's airway is occluded. A health care provider's order is not required to determine safe cuff pressure. A nonfenestrated inner cannula must be used to suction a patient to prevent tracheal damage occurring from the suction catheter passing through the fenestrated openings.

Which action will be included in the plan of care when the nurse is caring for a patient who is receiving nicardipine (Cardene) to treat a hypertensive emergency? (Exam 2) a. Organize nursing activities so that the patient has undisturbed sleep for 8 hours at night. b. Keep the patient NPO to prevent aspiration caused by nausea and possible vomiting. c. Assist the patient up in the chair for meals to avoid complications associated with immobility. d. Use an automated noninvasive blood pressure machine to obtain frequent measurements

d. Use an automated noninvasive blood pressure machine to obtain frequent measurements

A patient with acute dyspnea is scheduled for a spiral computed tomography (CT) scan. Which information obtained by the nurse is a priority to communicate to the health care provider before the CT? (Exam 1) a. O2 saturation of 90% b. Apical pulse of 104 c. Respiratory rate of 30 d. Allergy to shellfish

d. allergy to shellfish Response Feedback: Because iodine-based contrast media is used during a spiral CT, the patient may need to have the CT scan without contrast or be premedicated before injection of the contrast media. The increased pulse, low oxygen saturation, and tachypnea all indicate a need for further assessment or intervention but do not indicate a need to modify the CT procedure.

While caring for a 23-year-old patient with mitral valve prolapse (MVP) without valvular regurgitation, the nurse determines that discharge teaching has been effective when the patient states that it will be necessary to: (Exam 2) a. take antibiotics before any dental appointments. b. limit physical activity to avoid stressing the heart. c. take an aspirin a day to prevent clots from forming on the valve. d. avoid use of over-the-counter (OTC) medications that contain stimulant drugs

d. avoid use of over-the-counter (OTC) medications that contain stimulant drugs

A patient who uses a fentanyl (Duragesic) patch for chronic cancer pain suddenly complains of rapid onset pain at a level 9 (0 to 10 scale) and requests something for pain that will work now. How will the nurse document the type of pain reported by this patient? (Exam 6) a. Somatic pain b. Referred pain c. Neuropathic pain d. Breakthrough pain

d. breakthrough pain

A client is diagnosed with viral hepatitis, complaining of "no appetite" and "losing my taste for food." What instruction should the nurse give the client to provide adequate nutrition? (Exam 5)

increase intake of fluids, including juices

A 74-yr-old patient has just arrived in the emergency department. After assessment reveals a pulse deficit of 46 beats, the nurse will anticipate that the patient may require (Exam 2) a. emergent cardioversion. b. a cardiac catheterization. c. hourly blood pressure (BP) checks. d. electrocardiographic (ECG) monitoring.

d. electrocardiographic (ECG) monitoring.

A patient with suspected meningitis is scheduled for a lumbar puncture. Before the procedure, the nurse will plan to (Exam 3) a. enforce NPO status for 4 hours. b. transfer the patient to radiology. c. administer a sedative medication. d. help the patient to a lateral position

d. help the patient to a lateral position

A patient at the clinic says, "I always walk after dinner, but lately my leg cramps and hurts after just a few minutes of starting. The pain goes away after I stop walking, though." The nurse should (Exam 2) a. look for the presence of tortuous veins bilaterally on the legs. b. ask about any skin color changes that occur in response to cold. c. assess for unilateral swelling, redness, and tenderness of either leg. d. palpate for the presence of dorsalis pedis and posterior tibial pulses

d. palpate for the presence of dorsalis pedis and posterior tibial pulses

Nadolol (Corgard) is prescribed for a patient with chronic stable angina and left ventricular dysfunction. To determine whether the drug is effective, the nurse will monitor for (exam 2) a. decreased blood pressure and heart rate. b. fewer complaints of having cold hands and feet. c. improvement in the strength of the distal pulses. d. participation in daily activities without chest pain.

d. participation in daily activities without chest pain.

A client newly diagnosed with diabetes mellitus is instructed by the primary health care provider to obtain glucagon for emergency home use. The client asks the home care nurse about the purpose of the medication. The nurse should instruct the client that the purpose of the medication is to treat which problem? (Exam 5)

hypoglycemia from insulin overdose

The nurse has identified a nursing diagnosis of acute pain related to inflammatory process for a patient with acute pericarditis. The priority intervention by the nurse for this problem is to (Exam 2) a. teach the patient to take deep, slow breaths to control the pain. b. force fluids to 3000 mL/day to decrease fever and inflammation. c. remind the patient to request opioid pain medication every 4 hours. d. place the patient in Fowler's position, leaning forward on the overbed table.

d. place the patient in Fowler's position, leaning forward on the overbed table.

The nurse has received change-of-shift report about the following patients on the progressive care unit. Which patient should the nurse see first? (Exam 2) a. A patient who is in a sinus rhythm, rate 98, after having electrical cardioversion 2 hours ago b.A patient with new onset atrial fibrillation, rate 88, who has a first dose of warfarin (Coumadin) due c.A patient with second-degree atrioventricular (AV) block, type 1, rate 60, who is dizzy when ambulating d.A patient whose implantable cardioverter-defibrillator (ICD) fired two times today who has a dose of amiodarone (Cordarone) due

d.A patient whose implantable cardioverter-defibrillator (ICD) fired two times today who has a dose of amiodarone (Cordarone) due

A client who had abdominal surgery is receiving patient-controlled analgesia intravenously to manage pain. The pump is programmed to deliver a basal dose and bolus doses that can be accessed by the client, with a lock-out time frame of 10 minutes. The nurse assesses use of the pump during the last hour and identifies that the client attempted to self-administer the analgesic 10 times. Further assessment reveals that the client is experiencing pain still. What should the nurse do first? (Exam 5)

determine the integrity of the intravenous delivery system

A patient with late-stage cirrhosis develops portal hypertension. Which complications can develop from this condition? (select all that apply) (Exam 5)

esophageal varices ascites

After surgery for cancer of the pancreas, the client's nutrition and fluid regimen are influenced by the remaining amount of functioning pancreatic tissue. The nurse considers both the exocrine and the endocrine functions of the pancreas and expects that, postoperatively, the client's dietary regimen will be focused on the management of what substances? (Exam 5)

fats and carbs

The patient diagnosed with pancreatic cancer underwent a Whipple procedure 2 days ago. Which clinical problem has the highest priority? (Exam 5)

fluid volume imbalance

Metformin is prescribed for a client with type 2 diabetes mellitus. What is the most common side effect that the nurse should include in the client's teaching plan? (Exam 5)

flushing and palpitations

Cyclobenzaprine is prescribed to treat muscle spasms. Which disorder would indicate a need to notify HCP? (Exam 4)

glaucoma

A primary healthcare provider diagnoses a client with acute cholecystitis with biliary colic. Which clinical findings should the nurse expect when performing a health history and physical assessment? Select all that apply (Exam 5)

intolerance to foods high in fat, pain that radiates to the right shoulder

Which intervention would the nurse expect for a patient admitted with acute pancreatitis? (Exam 5)

keep the patient NPO

A patient has scleroderma manifested by CREST (calcinosis, Raynaud's phenomenon, esophageal dysfunction, sclerodactyly, telangiectasia) syndrome. Which action is included in plan of care? (Exam 4)

keeping the environment warm and draft free

Colchicine is prescribed for a client with a diagnosis of gout. The nurse review the client's record, knowing that this medication would be used with caution in which disorder? (Exam 4)

kidney disease

A patient is being discharged 4 day after hip arthroplasty, using the posterior approach. Which patient action requires intervention by the nurse? (Exam 4)

leaning over to pull on shoes and socks

The nurse is providing discharge instructions to a client following gastrectomy and should instruct the client to take which measure to assist in preventing dumping syndrome? (Exam 5)

limit the fluids taken with meals

The nurse is analyzing the lab studies on a client receiving dantrolene to treat muscle spasms from an injury. Which lab test identifies an adverse effect associated with this medication? (Exam 4)

liver function tests

While undergoing a soapsuds enema, the client reports abdominal cramping. What action should the nurse take? (Exam 5)

lower the height of the enema bag

A client with hiatal hernia chronically experiences heartburn following meals. The nurse should plan to teach the client to avoid which action because it is contraindicated with a hiatal hernia? (Exam 5)

lying recumbent following meals

A nurse is caring for a client with a suspected endocrine tumor that presents with hypertension. Which study will the nurse prepare to monitor that best screens for this condition? (Exam 5)

metanephrine

A client admitted to the hospital with chest pain and a history of type 2 diabetes mellitus is scheduled for cardiac catheterization. Which medication would need to be withheld for 24 hours before the procedure and for 48 hours after the procedure

metformin

A nurse is caring for a client who has a Hemovac portable wound suction device after abdominal surgery. What is the reason why the nurse empties the device when it is half full? (Exam 5)

negative pressure in the unit lessens as fluid accumulates, interfering with further drainage

The nurse is conducting teaching to a client in an arm cast about the S/S of compartment syndrome. The nurse determines the client understands the information if they report which early symptom of compartment syndrome? (Exam 4)

numbness and tingling in the fingers

The nurse is assessing a client who is experiencing an acute episode of cholecystitis. Which of these clinical manifestations support this diagnosis? Select all that apply (Exam 5)

pain in the upper right quadrant after a fatty meal fever complaints of indigestion

The nurse is reviewing the laboratory results for a client with cirrhosis and notes that the ammonia level is 85 mcg/dL (51 mcmol/L). Which dietary selection does the nurse suggest to the client? (Exam 5)

pasta with sauce

The nurse is caring for a client who had an above the knee amputation 2 days ago. The residual limb was wrapped with an elastic compression bandage which has come off. What do you do? (Exam 4)

rewrap the residual limb with an elastic compression bandage

A client is admitted to the hospital with severe diarrhea, abdominal cramps, and vomiting after eating. These symptoms have lasted 5 days. Upon further assessment, the primary healthcare provider finds that the symptoms occurred after the client ate eggs, salad dressings, and sandwich fillings. Which food borne disease would be suspected in this client? (Exam 5)

salmonellosis

A client with diabetes mellitus has had a right below the knee amputation. Given the client's history of diabetes mellitus, which complication is the client at most risk for after the surgery? (Exam 4)

separation of the wound edges

The nurse anticipates the preferred treatment for a patient with acute hepatitis A infection will include (Exam 5)

supportive care

A patient has had a surgical reduction of an open fracture of the right radius. Which finding should be immediately reported to the HCP? (Exam 4)

temperature of 101.4F

The nurse is caring for a client who has had a spinal fusion with insertion of hardware. The nurse would be most concerned with which assessment finding? (Exam 4)

temperature of 101.6F

The nurse is evaluating the client who is in skeletal traction. When evaluation the pin sites, the nurse would be most concerned with which finding? (Exam 4)

thick, yellow drainage from the pin sites

A home care nurse is visiting a client with a diagnosis of pernicious anemia that developed as a result of gastric surgery. The nurse instructs the client that in this disorder, because the stomach lining produces a decreased amount of a substance known as the intrinsic factor, the client will need which medication? (Exam 5)

vitamin B12 injections

A client with biliary cirrhosis receives serum albumin therapy. What is the most effective method for the nurse to evaluate the client's response to therapy? (Exam 5)

weigh daily

A person needs postexposure prophylaxis for hepatitis B with hepatitis B immune globulin (HBIG). When should HBIG be given after exposure? (Exam 5)

within 24 hours


Set pelajaran terkait

Chapter 11: Marketing: Helping Buyers Buy

View Set

Honors Chem The Ideal Gas Law Quiz

View Set

Homework #2 - Part 1 (automatically graded)

View Set

Digestive System Short Answer Questions

View Set

Foster positive relationships with customers to enhance company image

View Set

Religion (Chapter 10: The Communion of Saints)

View Set

CRM2280-001 Intro of Criminology Exam 2

View Set